Помощь студентам в учёбе от Людмилы Фирмаль
Здравствуйте!
Я, Людмила Анатольевна Фирмаль, бывший преподаватель математического факультета Дальневосточного государственного физико-технического института со стажем работы более 17 лет. На данный момент занимаюсь онлайн обучением и помощью по любыми предметам. У меня своя команда грамотных, сильных бывших преподавателей ВУЗов. Мы справимся с любой поставленной перед нами работой технического и гуманитарного плана. И не важно: она по объёму на две формулы или огромная сложно структурированная на 125 страниц! Нам по силам всё, поэтому не стесняйтесь, присылайте.
Срок выполнения разный: возможно онлайн (сразу пишите и сразу помогаю), а если у Вас что-то сложное – то от двух до пяти дней.
Для качественного оформления работы обязательно нужны методические указания и, желательно, лекции. Также я провожу онлайн-занятия и занятия в аудитории для студентов, чтобы дать им более качественные знания.
Моё видео:
youtube.com/embed/4sKc0lcxi7g» frameborder=»0″ allow=»accelerometer; autoplay; encrypted-media; gyroscope; picture-in-picture» allowfullscreen=»»/>
Вам нужно написать сообщение в Telegram . После этого я оценю Ваш заказ и укажу срок выполнения. Если условия Вас устроят, Вы оплатите, и преподаватель, который ответственен за заказ, начнёт выполнение и в согласованный срок или, возможно, раньше срока Вы получите файл заказа в личные сообщения.
Сколько может стоить заказ?Стоимость заказа зависит от задания и требований Вашего учебного заведения. На цену влияют: сложность, количество заданий и срок выполнения. Поэтому для оценки стоимости заказа максимально качественно сфотографируйте или пришлите файл задания, при необходимости загружайте поясняющие фотографии лекций, файлы методичек, указывайте свой вариант.
Какой срок выполнения заказа?Минимальный срок выполнения заказа составляет 2-4 дня, но помните, срочные задания оцениваются дороже.
Сначала пришлите задание, я оценю, после вышлю Вам форму оплаты, в которой можно оплатить с баланса мобильного телефона, картой Visa и MasterCard, apple pay, google pay.
Какие гарантии и вы исправляете ошибки?В течение 1 года с момента получения Вами заказа действует гарантия. В течении 1 года я и моя команда исправим любые ошибки в заказе.
Качественно сфотографируйте задание, или если у вас файлы, то прикрепите методички, лекции, примеры решения, и в сообщении напишите дополнительные пояснения, для того, чтобы я сразу поняла, что требуется и не уточняла у вас. Присланное качественное задание моментально изучается и оценивается.
Теперь напишите мне в Telegram или почту и прикрепите задания, методички и лекции с примерами решения, и укажите сроки выполнения. Я и моя команда изучим внимательно задание и сообщим цену.
Если цена Вас устроит, то я вышлю Вам форму оплаты, в которой можно оплатить с баланса мобильного телефона, картой Visa и MasterCard, apple pay, google pay.
Мы приступим к выполнению, соблюдая указанные сроки и требования. 80% заказов сдаются раньше срока.
После выполнения отправлю Вам заказ в чат, если у Вас будут вопросы по заказу – подробно объясню. Гарантия 1 год. В течении 1 года я и моя команда исправим любые ошибки в заказе.
youtube.com/embed/g4ioDc7sLwU» frameborder=»0″ allow=»accelerometer; autoplay; encrypted-media; gyroscope; picture-in-picture» allowfullscreen=»»/>
youtube.com/embed/zW5JhiZ4140″ frameborder=»0″ allow=»accelerometer; autoplay; encrypted-media; gyroscope; picture-in-picture» allowfullscreen=»»/>
Можете смело обращаться к нам, мы вас не подведем. Ошибки бывают у всех, мы готовы дорабатывать бесплатно и в сжатые сроки, а если у вас появятся вопросы, готовы на них ответить.
В заключение хочу сказать: если Вы выберете меня для помощи на учебно-образовательном пути, у вас останутся только приятные впечатления от работы и от полученного результата!
Жду ваших заказов!
С уважением
Пользовательское соглашение
Политика конфиденциальности
Помощь студентам в учёбе от Людмилы Фирмаль
Здравствуйте!
Я, Людмила Анатольевна Фирмаль, бывший преподаватель математического факультета Дальневосточного государственного физико-технического института со стажем работы более 17 лет. На данный момент занимаюсь онлайн обучением и помощью по любыми предметам. У меня своя команда грамотных, сильных бывших преподавателей ВУЗов. Мы справимся с любой поставленной перед нами работой технического и гуманитарного плана. И не важно: она по объёму на две формулы или огромная сложно структурированная на 125 страниц! Нам по силам всё, поэтому не стесняйтесь, присылайте.
Срок выполнения разный: возможно онлайн (сразу пишите и сразу помогаю), а если у Вас что-то сложное – то от двух до пяти дней.
Для качественного оформления работы обязательно нужны методические указания и, желательно, лекции. Также я провожу онлайн-занятия и занятия в аудитории для студентов, чтобы дать им более качественные знания.
Моё видео:
Вам нужно написать сообщение в Telegram . После этого я оценю Ваш заказ и укажу срок выполнения. Если условия Вас устроят, Вы оплатите, и преподаватель, который ответственен за заказ, начнёт выполнение и в согласованный срок или, возможно, раньше срока Вы получите файл заказа в личные сообщения.
Сколько может стоить заказ?Стоимость заказа зависит от задания и требований Вашего учебного заведения. На цену влияют: сложность, количество заданий и срок выполнения. Поэтому для оценки стоимости заказа максимально качественно сфотографируйте или пришлите файл задания, при необходимости загружайте поясняющие фотографии лекций, файлы методичек, указывайте свой вариант.
Минимальный срок выполнения заказа составляет 2-4 дня, но помните, срочные задания оцениваются дороже.
Как оплатить заказ?Сначала пришлите задание, я оценю, после вышлю Вам форму оплаты, в которой можно оплатить с баланса мобильного телефона, картой Visa и MasterCard, apple pay, google pay.
Какие гарантии и вы исправляете ошибки?В течение 1 года с момента получения Вами заказа действует гарантия. В течении 1 года я и моя команда исправим любые ошибки в заказе.
Качественно сфотографируйте задание, или если у вас файлы, то прикрепите методички, лекции, примеры решения, и в сообщении напишите дополнительные пояснения, для того, чтобы я сразу поняла, что требуется и не уточняла у вас. Присланное качественное задание моментально изучается и оценивается.
Теперь напишите мне в Telegram или почту и прикрепите задания, методички и лекции с примерами решения, и укажите сроки выполнения. Я и моя команда изучим внимательно задание и сообщим цену.
Если цена Вас устроит, то я вышлю Вам форму оплаты, в которой можно оплатить с баланса мобильного телефона, картой Visa и MasterCard, apple pay, google pay.
Мы приступим к выполнению, соблюдая указанные сроки и требования. 80% заказов сдаются раньше срока.
После выполнения отправлю Вам заказ в чат, если у Вас будут вопросы по заказу – подробно объясню. Гарантия 1 год. В течении 1 года я и моя команда исправим любые ошибки в заказе.
youtube.com/embed/NXwmZ4aXXsQ» frameborder=»0″ allow=»accelerometer; autoplay; encrypted-media; gyroscope; picture-in-picture» allowfullscreen=»»/>
youtube.com/embed/WIbBf3NZEZI» frameborder=»0″ allow=»accelerometer; autoplay; encrypted-media; gyroscope; picture-in-picture» allowfullscreen=»»/>
youtube.com/embed/dihxr1liTvQ» frameborder=»0″ allow=»accelerometer; autoplay; encrypted-media; gyroscope; picture-in-picture» allowfullscreen=»»/>
Можете смело обращаться к нам, мы вас не подведем. Ошибки бывают у всех, мы готовы дорабатывать бесплатно и в сжатые сроки, а если у вас появятся вопросы, готовы на них ответить.
В заключение хочу сказать: если Вы выберете меня для помощи на учебно-образовательном пути, у вас останутся только приятные впечатления от работы и от полученного результата!
Жду ваших заказов!
С уважением
Пользовательское соглашение
Политика конфиденциальности
Помощь студентам в учёбе от Людмилы Фирмаль
Здравствуйте!
Я, Людмила Анатольевна Фирмаль, бывший преподаватель математического факультета Дальневосточного государственного физико-технического института со стажем работы более 17 лет. На данный момент занимаюсь онлайн обучением и помощью по любыми предметам. У меня своя команда грамотных, сильных бывших преподавателей ВУЗов. Мы справимся с любой поставленной перед нами работой технического и гуманитарного плана. И не важно: она по объёму на две формулы или огромная сложно структурированная на 125 страниц! Нам по силам всё, поэтому не стесняйтесь, присылайте.
Срок выполнения разный: возможно онлайн (сразу пишите и сразу помогаю), а если у Вас что-то сложное – то от двух до пяти дней.
Для качественного оформления работы обязательно нужны методические указания и, желательно, лекции. Также я провожу онлайн-занятия и занятия в аудитории для студентов, чтобы дать им более качественные знания.
Моё видео:
Вам нужно написать сообщение в Telegram . После этого я оценю Ваш заказ и укажу срок выполнения. Если условия Вас устроят, Вы оплатите, и преподаватель, который ответственен за заказ, начнёт выполнение и в согласованный срок или, возможно, раньше срока Вы получите файл заказа в личные сообщения.
Стоимость заказа зависит от задания и требований Вашего учебного заведения. На цену влияют: сложность, количество заданий и срок выполнения. Поэтому для оценки стоимости заказа максимально качественно сфотографируйте или пришлите файл задания, при необходимости загружайте поясняющие фотографии лекций, файлы методичек, указывайте свой вариант.
Какой срок выполнения заказа?Минимальный срок выполнения заказа составляет 2-4 дня, но помните, срочные задания оцениваются дороже.
Как оплатить заказ?Сначала пришлите задание, я оценю, после вышлю Вам форму оплаты, в которой можно оплатить с баланса мобильного телефона, картой Visa и MasterCard, apple pay, google pay.
В течение 1 года с момента получения Вами заказа действует гарантия. В течении 1 года я и моя команда исправим любые ошибки в заказе.
Качественно сфотографируйте задание, или если у вас файлы, то прикрепите методички, лекции, примеры решения, и в сообщении напишите дополнительные пояснения, для того, чтобы я сразу поняла, что требуется и не уточняла у вас. Присланное качественное задание моментально изучается и оценивается.
Теперь напишите мне в Telegram или почту и прикрепите задания, методички и лекции с примерами решения, и укажите сроки выполнения. Я и моя команда изучим внимательно задание и сообщим цену.
Если цена Вас устроит, то я вышлю Вам форму оплаты, в которой можно оплатить с баланса мобильного телефона, картой Visa и MasterCard, apple pay, google pay.
Мы приступим к выполнению, соблюдая указанные сроки и требования. 80% заказов сдаются раньше срока.
После выполнения отправлю Вам заказ в чат, если у Вас будут вопросы по заказу – подробно объясню.
youtube.com/embed/vOuj6t2Zyeg» frameborder=»0″ allow=»accelerometer; autoplay; encrypted-media; gyroscope; picture-in-picture» allowfullscreen=»»/>
youtube.com/embed/ElMEic2MJ-U» frameborder=»0″ allow=»accelerometer; autoplay; encrypted-media; gyroscope; picture-in-picture» allowfullscreen=»»/>
Можете смело обращаться к нам, мы вас не подведем. Ошибки бывают у всех, мы готовы дорабатывать бесплатно и в сжатые сроки, а если у вас появятся вопросы, готовы на них ответить.
В заключение хочу сказать: если Вы выберете меня для помощи на учебно-образовательном пути, у вас останутся только приятные впечатления от работы и от полученного результата!
Жду ваших заказов!
С уважением
Пользовательское соглашение
Политика конфиденциальности
3.2. Решение типовых задач по линейной алгебре
3. Задачи линейной алгебры | 41 |
last(V) – возвращает номер последней компоненты вектора V. length(V) – возвращает количество компонент вектора V. rows( A) – возвращает количество строк в матрице A. cols(A) – возвращает количество столбцов в матрице A. max(A, B, C, …) , min(A, B, C, …) – возвращают соответст-
венно максимальное и минимальное значение элементов матриц, включенных в скобки.
tr(A) – возвращает сумму диагональных элементов матрицы A (след матрицы A).
rank(A) – вычисляет ранг матрицы A.
norm1(A), norm2(A), norme(A), normi(A) – вычисление норм матрицы A по разным методам.
3. Функции, реализующие численные алгоритмы решения задач линейной алгебры.
eigenvals(A) – вычисление собственных значений квадратной матрицы A.
eigenvecs(A) – вычисление собственных векторов квадратной матрицы A.
lsolve(A,b) – решение системы линейных алгебраических уравнений Ax=b.
rref(A) – приведение матрицы к ступенчатому виду при помощи элементарных преобразований с единичным базисным минором.
Рассмотрим решение нулевого варианта средствами Mathcad, приведенного в работе [4].
41
Пример 1. Найти миноры M22, алгебраические дополнения A31 и вычислить определители матриц
| 5 | 6 | 3 |
|
| 1 | −2 | 3 | 4 |
| |
, B = 2 | 1 −4 | 3 . | |||||||||
A = | 0 | 1 | 0 |
| |||||||
|
|
|
|
|
| 3 | −4 | −1 | −2 |
| |
| 7 | 4 | 5 |
| |||||||
|
|
| 4 | 3 | 2 |
|
| ||||
|
|
|
|
|
| −1 |
Решение:
Все элементы одномерных и двумерных массивов будем нумеровать начиная с 1, поэтому вначале файла вставим команду
ORIGIN :=1
Чтобы ввести матрицу, необходимо набрать имя матрицы и знак присвоить. Затем на панели операций с матрицами нажать
на кнопку или использовать «горячую» комбинацию клавиш «Ctrl+m» (удерживая клавишу Ctrl, нажать другую клавишу m). В появившемся диалоговом окне необходимо ввести число строк (Rows) и число столбцов (Columns). Перемещаться к следующему элементу можно при помощи мышки, стрелок или клавиши Tab. Чтобы вычислить определитель матрицы A, необходимо нажать
кнопку или клавишу | и ввести имя матрицы A, и нажать клавишу пробела и знак =.
a) Решаем первую часть примера 1. Вводим матрицу A.
| 5 | 6 | 3 |
|
|
| 0 | 1 | 0 |
| . Для этого набираем следующую последователь- |
A := |
| ||||
| 7 | 4 | 5 |
|
|
|
|
|
ность символов: A «Shift+:» «Ctrl+m» 3 3 5 Tab 6 Tab 3 Tab 0 Tab 1 Tab 0 Tab 7 Tab 4 Tab 5 Enter.
Вычисляем определитель этой матрицы |A|=4.
Для вычисления минора M22 вычислим определитель
| 5 | 3 |
| = 4. |
| 7 | 5 |
| |
|
|
|
Аргумент определителя в данном случае быстрее ввести вручную, вставив и заполнив матрицу второго порядка. Для этого
мышкой нажимаем кнопку или клавишу | и вместо аргумента
42
3. Задачи линейной алгебры | 43 |
вводим матрицу второго порядка, используя кнопку , после чего нажимаем клавишу =.
Вычислим теперь алгебраическое дополнение A31. Для этого используем функцию submatrix.
A31:= (−1)1+3 | submatrix(A,1,2,2,3) |.
При помощи функции submatrix, из матрицы A порядка 3, извлекаем матрицу второго порядка, получающуюся вычеркиванием первого столбца и третьей строки. Вычисляем определитель из этой матрицы и умножаем его на знак алгебраического дополнения. Для вывода результата вычисления вводим имя переменной A31, в которой хранится результат, и нажимаем клавишу =. Получаем, A31= –3.
b) Теперь найдем те же величины для матрицы B.
Вводим переменную B1 и присваиваем ей значение, соответствующее квадратной матрице B, состоящей из четырех строк
| 1 | −2 | 3 | 4 |
|
| 2 | 1 | −4 | 3 |
|
(Rows) и четырех столбцов (Columns). B := | . | ||||
| 3 | −4 | −1 | −2 |
|
|
| ||||
| 4 | 3 | 2 | −1 |
|
Вычисляем определитель |B|=900.
Теперь вычислим минор элемента B22. Для этого получим матрицу B22, исключив из матрицы B вторую строку и второй столбец. Проще набрать вручную эту матрицу. С целью демонстрации использования матричных функций получим матрицу
B22 при помощи функций augment, stack и submatrix. Чтобы фор-
мула была понятнее, используем две вспомогательные матрицы B1 и B2. B1 – это первая строка без второго столбца. B2 – третья и четвертая строки без второго столбца.
B1:= augment(submatrix(B,1,1,1,1), submatrix(B,1,1,3,4)). B2 := augment(submatrix(B,3,4,1,1), submatrix(B,3,4,3,4)).
Затем, используя функцию stack, соединяем эти две матрицы в одну матрицу B22. B22 := stack(B1, B2).
| 1 | 3 | 4 |
|
| 3 | −1 | −2 |
|
Получили матрицу B22 = | . | |||
| 4 | 2 | −1 |
|
|
|
43
Теперь вычисляем определитель матрицы B22 и получаем минор B22. | B22 |= 30.
Вычислим теперь алгебраическое дополнение B31. Для того чтобы из матрицы B вычеркнуть третью строку и первый столбец, нужно вырезать из B два блока, имеющие по три столбца: submatrix(B,1,2,2,4) и submatrix(B,4,4,2,4), – присоединить к пер-
вому второй блок, используя функцию stack.
M 31:= stack(submatrix(B,1,2,2,4), submatrix(B,4,4,2,4)).
Далее вычисляем и запоминаем в переменной B31 значение алгебраического дополнения B31. B31:= (−1)3+1 | M 31| . Выводим значение переменной B31. B31= –124.
0 −1 5
Пример 2. Решить уравнение: 3 4 7 =12. x x 8
Решение: В общем случае такая задача решается разложением определителя на множители и решением полученного алгебраического уравнения. Раскладываем этот определитель по третьей строке. Получаем линейное уравнение:
x |
|
| −1 5 |
| − x |
| 0 | 5 |
| +8 |
| 0 | −1 |
| =12. | |||||
|
|
|
|
|
| |||||||||||||||
|
| 4 7 |
|
|
|
| 3 | 7 |
|
|
|
| 3 | 4 |
|
| ||||
|
|
|
|
|
|
|
|
|
|
|
|
|
|
|
|
Для решения этого уравнения мышкой выделяем любую из двух переменных x и переходим в пункт меню Simbolics, выбираем пункт подменю Variable и вызываем команду Solve. В результате получаем ответ: 1.
Для проверки правильности решения введем переменную x и присвоим ей значение x:=1. Затем, введем матрицу A, соответст-
|
| 0 | −1 | 5 |
|
|
вующую условию задачи. |
| 3 | 4 | 7 |
| . Вычислим определи- |
A := |
| |||||
|
|
| x | 8 |
|
|
| x |
|
|
тель матрицы A. |A|=12.
Для версий Mathcad, начиная с 2001, удобнее использовать вычислительный блок Given/Find.
44
3. Задачи линейной алгебры | 45 |
|
| 0 | −1 | 5 |
|
|
|
| |||||
x := 0 Given |
| 3 | 4 | 7 |
| =0 x := Find(x). |
|
| |||||
|
|
| x | 8 |
|
|
| x |
|
|
Осталось узнать значение искомой переменной x=1. Напомним, что знак = в уравнении набирается как знак логической операции. («Ctrl+=» или используя команду = в панели инструментов
Boolean).
Пример 3. Найти произведения матриц A B:
| 1 | 0 1 |
|
| 1 | 2 | 2 |
|
| 2 | −1 3 |
|
| 2 | 1 | −2 |
|
A = |
| B = | . | |||||
| 0 2 −1 |
|
| 2 | −2 1 |
| ||
|
|
|
|
Решение: Вводим с клавиатуры значения всех элементов перемножаемых матриц и набираем команду A*B=. В результате
|
| 3 | 0 | 3 |
|
получаем: |
| 6 | −3 | 9 |
|
A B = | . | ||||
|
| 2 | 4 | −5 |
|
|
|
|
Пример 4. Доказать, что матрица A имеет обратную и найти
её:
| 2 | 3 | 2 |
|
| 1 | 2 | −3 |
|
A := | . | |||
| 3 | 4 | 1 |
|
|
|
Решение. Вводим матрицу A. Для доказательства существования обратной матрицы, вычисляем её определитель. |A|= –6. Он не равен нулю, следовательно, существует обратная матрица A−1 . Для определения обратной матрицы вводим имя
матрицы A и нажимаем на кнопку . Затем два раза нажимаем клавишу пробел и клавишу =. В результате получаем обратную матрицу с точностью до трех знаков после запятой:
|
| −2.333 | −0.833 | 2.167 |
| |
A | -1 |
| 1.667 | 0.667 | −1.333 |
|
| = | . | ||||
|
|
| 0.333 | −0.167 | −0.167 |
|
|
|
|
|
Для того чтобы получить более точный ответ, необходимо дважды щелкнуть мышкой внутри поля обратной матрицы. В
45
появившемся окне Result Format выбрать поле Decimal и в области Number of Decimal places установить необходимое количество знаков после запятой.
| 1 | 0 | 2 | 0 | 0 |
|
| 0 | 1 | 0 | 2 | 0 |
|
Пример 5. Найти ранг матрицы: A := | . | |||||
| 2 | 0 | 4 | 0 | 0 |
|
|
|
Решение. Вводим матрицу A, состоящую из трех строк и пяти столбцов. Вызываем функцию rank(A) и нажимаем клавишу =.
Получаем ответ: rank(A) = 2.
Пример 6. Решить систему линейных уравнений: а) матричным методом; б) методом Крамера; в) методом Гаусса.
| 2x + x | + x | = | 2, |
|
|
|
|
|
|
| |||
|
| 1 | 2 | 3 |
|
|
|
|
|
|
|
|
| |
x1 |
| + x2 +3×3 | = 6, |
|
|
|
|
|
|
| ||||
2x |
| + x | + 2x | = | 5. |
|
|
|
|
|
|
| ||
| 1 |
| 2 | 3 |
|
|
| 2 | 1 | 1 |
|
|
| |
|
|
|
|
|
|
|
|
|
|
|
| |||
Решение. | а) | Вводим матрицу |
| 1 | 1 | 3 |
| и | вектор | |||||
A := |
| |||||||||||||
|
|
|
|
|
|
|
|
| 2 | 1 | 2 |
|
|
|
b := (2 |
|
|
| 5)T . |
|
|
|
| ||||||
| 6 | x := A−1 b. | Выводим | ответ, |
транспонируя вектор-столбец (с целью экономия места в учебном пособии): xT = (2 −5 3).
Проверяем полученный результат: (A x)T = (2 6 5). По-
лучили правую часть. Следовательно, система линейных алгебраических уравнений решена правильно.
б) Решаем систему методом Крамера. На базе матрицы A получаем три вспомогательные матрицы: A1, A2 и A3. В матрице A1, вместо первого столбца, в матрице A2, вместо второго столбца и в матрице A3, вместо третьего столбца, подставляем вектор b.
A1:= augment(b, A 2 , A 3 ), A2 := augment( A 1 ,b,A 3 ) и A3 := augment( A 1 ,A 2 ,b ).
Вычисляем определитель матрицы A. d:=|A|. Сохраняем его значение в переменной d. По формулам Крамера получаем все
46
3. | 47 |
три компоненты вектора решений x1, x2, x3 и, используя функцию stack, запоминаем их в векторе y.
| A1| | | A2 | | | A3| | |||||
y := stack |
|
|
|
|
| . | |
d | d | d | |||||
|
|
Осталось вывести полученное решение: yT = (2 −5 3).
в) Решение методом Гаусса. В Mathcad прямой и обратный ход метода Гаусса выполняет функция rref(A), где матрица A является расширенной матрицей. В первых столбцах матрицы находится левая часть системы линейных алгебраических уравнений, а в последних столбцах – правые части системы, которых может быть несколько. Данная функция при помощи элементарных преобразований приводит расширенную матрицу к матрице, содержащей в левом верхнем угле единичную подматрицу. При помощи функции rref можно решать системы линейных алгебраических уравнений с несколькими правыми частями, определять ранг матрицы, находить фундаментальную систему, решать системы уравнений, имеющие бесконечное множество решений, и получать обратную матрицу.
Для того чтобы получить обратную матрицу для квадратной матрицы A порядка N, необходимо к матрице A справа приписать единичную матрицу порядка N и для полученной расширенной матрицы состоящей из N строк и 2N столбцов, применить функцию rref. В результате получим матрицу, в которой слева будет единичная матрица, а справа – обратная матрица.
Пример:
1 | 2 | 1 0 1 −2 |
| |
G := | . | rref (augment,identify(2))) = | . |
|
0 | 1 | 0 1 0 | 1 |
|
Обратной матрицей для матрицы G будет матрица 1 | −2 | . | ||
|
| 0 | 1 |
|
Учитывая, что матрица A и вектор b уже определены ранее, решить пример 6 методом Гаусса можно одной командой
| 1 | 0 | 0 | 2 |
|
| 0 | 1 | 0 | −5 |
|
rref ( augment( A,b )) = | . | ||||
| 0 | 0 | 1 | 3 |
|
|
|
47
Последний столбец результирующей матрицы и есть решение нашей системы.
Эту же задачу можно решить при помощи функции lsolve(A,b), где A – квадратная матрица, b – вектор правых частей. Решение примера получается при помощи команды:
lsolve( A,b )T = (2 −5 3).
Пример 7. Установить совместимость систем уравнений и решить их, если они совместны:
| x + x + x | =3, |
|
| x + x + x =6, | |||||
|
| 1 | 2 | 3 |
|
|
| 1 | 2 | 3 |
а) | 2×1 + x2 −2×3 =1, | б) |
| x1 − x2 + 2×3 =5, | ||||||
| x | + 2x | −3x =1, |
| 2x − x + x =3, | |||||
|
|
|
| |||||||
| 1 | 2 |
| 3 |
| 1 | 2 | 3 | ||
| x + x + x | =−1; |
| 3x −6x +5x =6. | ||||||
|
| 1 | 2 | 3 |
|
|
| 1 | 2 | 3 |
Решение:
а) Выпишем расширенную матрицу (назовем ее B) данной
| 1 | 1 | 1 | 3 |
|
| 2 | 1 | −2 | 1 |
|
системы. B := | . | ||||
1 | 2 | −3 | 1 |
| |
1 | 1 | 1 | −1 |
|
Очевидно, что ранг исследуемой системы не больше количества неизвестных, т. е. не более трех. Подаем команду rref(B) =.
| 1 | 0 | 0 | 0 |
|
| 0 | 1 | 0 | 0 |
|
Получаем следующий результат: rref ( B ) = | . | ||||
| 0 | 0 | 1 | 0 |
|
|
| ||||
| 0 | 0 | 0 | 1 |
|
Отсюда следует, что ранг матрицы системы A равен трем, а ранг расширенной матрицы B равен четырем. Следовательно, согласно теореме Кронекера–Капели система несовместна.
б) Выпишем расширенную матрицу (назовем ее B) данной
|
| 1 | 1 | 1 | 6 |
|
|
|
| 1 | −1 | 2 | 5 |
|
|
системы | B := | . | Применяем к этой матрице функ- | ||||
|
| 2 | −1 | 1 | 3 |
|
|
|
|
|
| ||||
|
| 3 | −6 | 5 | 6 |
|
|
цию rref:
48
3. Задачи линейной алгебры | 49 |
| 1 | 0 | 0 | 1 |
|
| 0 | 1 | 0 | 2 |
|
rref ( B ) = | . | ||||
| 0 | 0 | 1 | 3 |
|
|
| ||||
| 0 | 0 | 0 | 0 |
|
Ранг исследуемой системы и ранг расширенной матрицы равен трем. Таким образом, система уравнений совместна и имеет единственное решение. После применения элементарных преобразований получили систему трех уравнений. Четвертое уравнение превратилось в тождество.
Ответ: x1=1, x2=2, x3=3.
Задание для самостоятельной работы.
Выполнить свой вариант типового задания по линейной алгебре из сборника типовых расчетов [4].
49
Задача линейной алгебры — Энциклопедия по экономике
ML, транспонированная по отношению к А = [а.] есть М. того же размера, у которой столбцы поменялись местами со строками. Иначе говоря, это [я.
Функции, реализующие численные алгоритмы решения задач линейной алгебры [c.52]
При решении задач линейной алгебры необходимость вычислять определитель возникает достаточно часто. Наиболее распространенные приложения определителя — исследование и решение линейных систем. Исследованию линейных систем посвящен специальный раздел. Здесь рассмотрим решение линейных систем по формулам Крамера. Пусть [c.64]
Задачи линейной алгебры. Это вопросы векторных пространств и линейных преобразований в них, решение линейных уравнений, действия над векторами, операции над матрицами, тензорное исчисление. [c.81]
Пакет представляет собой совокупность подпрограмм, предназначенных для решения задач линейной алгебры, численного анализа и математической статистики. Он состоит из 132 подпрограмм, 9 программ-примеров и 8 специальных подпрограмм, необходимых для работы программ-примеров.
[c.183]
Задачи по оптимизации решаются различными математическими методами. В основе современных математических методов, применяемых в планировании работы предприятий, лежат следующие разделы математики теория вероятностей и математическая статистика, линейная алгебра и матричное исчисление. Применение теории вероятностей при планировании сложных систем и массовых явлений в геологоразведочном, буровом и нефтегазодобывающем производствах связано с необходимостью устанавливать не результат отдельных событий, а общий результат всей массы событий, при этом объектом планирования является многозначная, вероятностная система связей, а не строго определенная однозначная связь. Такие связи существуют, например, между производительностью скважин и многими производственными факторами, включая объем нагнетания воды в пласт, природные факторы производительности, режим работы скважин и др.

Линейная алгебра лежит в основе методов линейного программирования. С помощью этих методов могут быть решены следующие задачи рассчитана производственная мощность предприятия, цехов, участков разработаны производственные задания цехам, участкам и др. [c.153]
Задачи по оптимизации решаются различными математическими методами, в основе которых лежат теория вероятностей и математическая статистика, линейная алгебра, нелинейное программирование и, в частности, его простейшая форма — квадратичное программирование, а также стохастическое и динамическое программирования и, наконец, матричное исчисление. [c.18]
Для решения такой простой задачи, как эта, не нужны ни алгебра, ни геометрия. Решение является очевидным для тех, кто знает, что нужно искать при решении задачи линейного программирования. В данном случае ограниченным ресурсом является время, необходимое для изготовления продукции, дающей максимальную валовую прибыль за единицу производственного времени и возможный объем продаж. Наибольшую валовую прибыль за единицу производственного времени дает продукт А.
[c.218]
Сборник задач по математике для втузов. Ч. 1 Линейная алгебра и основы [c.144]
При решении многих задач в математике и ее приложениях приходится оперировать многомерными объектами, рассматривать их линейные комбинации и т.п. Методы адекватного описания таких объектов и соотношений между ними были разработаны математиками в рамках векторного и матричного исчисления, а также линейной алгебры. Область применения векторного и матричного исчисления расширилась, когда оказалось, что решение многих нелинейных задач достигается путем линеаризации. Примерами этого могут служить приближенный метод Ньютона для определения корней уравнения, а также линеаризация результатов измерений, первоначально подчиняющихся экспоненциальной или степенной закономерности, с последующей линейной аппроксимацией. [c.47]
Проведен численный анализ зависимости ускорения, достигаемого при распараллеливании явного метода решения системы нелинейных динамических систем от параметров ВС — числа процессоров и скорости работы каналов обмена данными. Так как веса всех вершин и дуг графа этого алгоритма суть величины одного порядка по N (где N есть размерность задачи), то увеличение N, в отличие от рассмотренных выше алгоритмов линейной алгебры, на ускорение никак не влияет. По причине больших объемов передаваемых данных при выполнении алгоритма ускорение больше 1 достигается только на очень высоких (относительно производительности процессоров) скоростях каналов, что делает рассмотренный метод мало пригодным к выполнению на большинстве реальных
[c.166]
Другим важным обстоятельством, определяющим неклассический характер задачи оптимального управления, является наличие в задаче условий типа неравенств. Это — условия и (t) U, условия (17), (18). Они, как показал опыт решения таких задач, весьма существенны снятие подобных условий обычно полностью лишает задачу содержательной ценности, так как приводит к решениям либо физически нелепым, либо неприемлемым по техническим условиям. Как правило, в оптимальном решении имеются как интервалы времени, на которых реализуется знак равенства, так и интервалы, на которых реализуется строгое неравенство на первых условие может быть заменено привычным для классического вариационного исчисления условием типа равенства, на последних — снято. К сожалению, расположение и размеры этих интервалов выясняются лишь после решения задачи. Это обстоятельство также имеет глубокие последствия в вопросах конструирования численных методов классический вычислительный аппарат линейной алгебры становится неэффективным и заменяется более соответствующим характеру современных вариационных задач вычислительным аппаратом линейного (и нелинейного) программирования.
[c.29]
Собственно вычислительный аппарат алгоритма должен быть адекватен задаче. Мы имеем дело с неклассической задачей, в условия которой входят неравенства. Поэтому привычный вычислительный аппарат линейной алгебры, ориентированный на решение задач в терминах равенств, недостаточен, следует привлечь аппарат линейного программирования. Этим работа автора существенно отличается от основной массы алгоритмов, которые так или иначе связаны с привычным аппаратом линейной алгебры. [c.113]
В заключении данного раздела следует описать еще один вид матриц (и векторов), которые обычно не рассматриваются в стандартных курсах по линейной алгебре или рассматриваются, но вскользь. В то же время, они имеют прямое отношение к экономическим и бухгалтерским задачам, — это матрицы с окаймляющими итоговыми строками и столбцами. Их, чтобы отличать от обычных матриц без итоговых строк и столбцов, будем для краткости называть окаймленными матрицами и векторами.
[c.372]
Гораздо сложнее задачи на отыскание оптимума при многих неизвестных (xl,x2,…, хп). Такие задачи часто возникают в оперативной и особенно в плановой практике и сводятся к отысканию оптимального варианта плана распределения наличных ресурсов по направлениям их возможного использования. Падежным методом решения этих задач является линейное программирование, представляющее собой прямое развитие нек-рых разделов линейной алгебры. [c.415]
Метод симплекс (8.1.) — метод решения основной задачи линейного программирования, заключающийся в целенаправленном переборе вершин многогранника ограничений с помощью методов линейной алгебры. [c.344]
В качестве инструментария для решения тестовых задач необходимы знания основных разделов математического программирования, эконометрии, линейной алгебры и математического анализа.
[c.159]
В справочник включены следующие разделы линейная алгебра, математическое программирование, теория графов и ее приложение, теория вероятностей, статистический анализ и др».,— широко использующиеся в экономических исследованиях и позволяющие проводить различного рода расчеты при решении экономических задач. [c.2]
Большую роль в экономическом моделировании играют методы математического программирования и сетевого планирования, опирающиеся на линейную алгебру, анализ функций одной и многих переменных и некоторые другие разделы математики. Растет роль теории игр при решении экономических задач. Значительное место в экономических исследованиях занимают статистические методы. По всем этим вопросам имеется много специальной литературы, изучение которой требует серьезной математической подготовки. то же время ощущается потребность в литературе, содержащей информацию из указанных разделов в достаточно обозримой и доступной форме. [c.3]
К задачам прямой обработки данных принято относить такие автоматизируемые планово-экономические задачи, реализация которых не требует применения специальных математических методов решения. В отличие от задач, базирующихся на экономико-математических моделях, в решении которых используются методы матричной алгебры, линейного программирования, математической статистики и другие, задачи прямой обработки данных сводятся к обработке на ЭВМ больших массивов информации при помощи простейших алгоритмов сортировки, табулирования, агрегирования и других, а также преобразований по элементарным формулам (например, потребность в данном ресурсе на производство какой-либо продукции определяется как произведение соответствующей удельной нормы расхода на объем производства этой продукции).
[c.126]
При качественном анализе систем в уравнении 5.2.3 оставляют только линейные эффекты. При большом числе факторов, которые необходимо учитывать, для предварительного заключения об их значимости используют методы алгебры логики. Исследование обычно проводится в два этапа. На первом этапе рабочие диапазоны изменения переменных квантуются на отдельные уровни и методом минимизации булевых функций строится булева модель системы. На втором этапе решается задача содержательной интерпретации булевых моделей.
[c.221]
Этот раздел является одним из основных в алгебре При решении экономических задач системы линейных уравнении наиболее употреби мы аппарате исследования и при рассмотрении частных проблем. [c.32]
Функции, предназначенные для решения задач линейной алгебры, собраны в разделе Векторы и матрицы (Ve tor and Matrix) их можно разделить на три группы функции определения матриц и операции с блоками матриц, функции вычисления различных числовых характеристик матриц и функции, реализующие численные алгоритмы решения задач линейной алгебры. Опишем наиболее часто используемые функции. [c.50]
При решении некоторых задач в линейной алгебре возникает задача определения собственных чисел и соответствующих им собственных функций, операторов с блочно-трехдиагональной матрицей L. Сложность задачи определяется, как правило, плохой обусловленностью матрицы, когда максимальное и минимальное собственные числа отличаются на несколько порядков. Для решения этой задачи используется метод обратной итерации.
[c.161]
В данной задаче допустимые по условиям (21) fo образуют линейное пространство (в частности, если fo удовлетворяет (21), то и —fo удовлетворяет (21), и нам достаточно найти любое fo, удовлетворяющее (21), для которого /° (х) fo= 0). Таким образом, оптимальной (неулучшаемой) точкой х может быть только такая, в которой из fx (х) fo=0, i=l, 2,. . ., 771 следует /° (х) fo=0. В линейной алгебре установлено, что это эквивалентно линейной зависимости градиентов f x (х), т. е. должны существовать множители Хх, А2,. . ., т такие, что [c.399]
Теперь, когда увеличиваются темпы роста производительных сил и разделения общественного труда, расширяются кооперирование и (внутрихозяйственные связи предприятий, что приводит к появлению все большего числа взаимозависимых переменных, выявление (которых возможно только с использованием математических методов, подтверждается известное высказывание К. Маркса о том, что наука только тогда достигает совершенства, когда ей удается пользоваться математикой . Таким образом, применение математики в экономике важно как для решения практических задач, так и для ее тео- ретического развития и превращения в точную науку. Необходимая точность в решении экономических задач, и особенно нахождение оптимальных вариантов проектироваийя, изготовления монтажа и эксплуатации кислородных производств, возможна только с использованием линейной алгебры, дифференциального и интегрального исчислений, теории вероятностей, математической статистики, математического программирования, теории массового обслуживания, сетевого анализа и других математических методов, которые будут рассмотрены ниже.
[c.177]
Из методов линейной алгебры, используемых в эко-номич. исследованиях, следует прежде всего указать на матричное исчисление, при помощи к-рого подвергаются строгому математич. анализу межотраслевые и межрайонные хоз. связи. Разработка такого метода анализа, начатая в СССР академиком В. С. Немчиновым, создает базу для применения теории воспроизводства К. Маркса в конкретных хозяйственно-исторнч. условиях. Эти работы связаны также с реализацией в хоз. практике закона пропорционального развития нар. х-ва. Они сопровождаются решением таких важнейших задач, как определение полных затрат общественного труда и различных материальных ресурсов на произ-во разных видов продукции, установление взаимоувязанной системы цен и др. Применение матричной алгебры к экономич. исследованиям предполагает широкое использование вычислительной математики и быстродействующей электронной счетной техники.
[c.415]
В первом варианте число аналогов равно количеству ценообра-зующих факторов, увеличенному на единицу. Для решения задач такого рода используется математический аппарат линейной алгебры. [c.280]
Теорию арбитража формально можно изобразить через лемму Минковского—Фаркаша. Эта теорема разделения содержит четкие критерии для различения между рынками капитала с существованием и без существования возможностей арбитража. Характерным для свободы от арбитража является существование ценового вектора как линейной комбинации линейно независимых векторов. Если этой линейной комбинации не существует, то возможны арбитражные прибыли. Мы хотим изобразить лемму графически и вынуждены для этой цели провести некоторую подготовительную работу . Первая задача — познакомиться с необходимыми аспектами векторной алгебры. На основе этого мы соединим формальные выводы леммы с уже полученными знаниями из обоих предыдущих разделов этой главы.
[c.133]
Вальрас Л. (1834—1910) — швейцарский экономист, основатель лозаннской школы, разработавший математический метод, основанный на широком использовании алгебры для исследования экономических явлений и процессов, которые рассматривал как функции, переменные или производные величины, как задачи из теории множества и матричной алгебры. Наряду с австрийской (К. Менгер) и англо-американской (А. Маршалл) школами он заложил фундамент современной экономической теории. Предложил концепцию общего экономического равновесия как универсального средства анализа экономической системы в целом, в основе которой лежало представление об экономическом поведении как об индивидуальной оптимизации. Пришел к теории общего равновесия, в которой рынки всех товаров оказываются взаимосвязанными все цены благ, выпуски товаров и наличие ресурсов взаимно и одновременно согласовываются в процессе функционирования экономики, а все ее секторы и участники стремятся максимизировать полезность. Его система была усовершенствована многочисленными последователями, в частности она была преобразована средствами линейного программирования в экстремальную задачу на оптимум. Сделал решительный шаг в сторону математизации экономической тео-
[c.9]
Примеры решения задач типового расчёта по линейной алгебре и аналитической геометрии
Примеры решения задач типового расчёта
по линейной алгебре и аналитической геометрии.
Задача 1.1. Вычислить, представить ответ в алгебраической, тригонометрической, показательной формах.
Решение. Представим комплексное число в тригонометрической форме. Для этого находится его модуль: , далее вычисляется аргумент комплексного числа: , либо , наконец,
, где , если комплексное число находится в первой
или четвертой четверти, , если комплексное число
находится во второй или третьей четверти. В данном случае аргумент комплексного
числа равен: .
Поэтому тригонометрическая форма имеет вид: .
По формуле Муавра , где — аргумент комплексного числа , получаем:
,
окончательно: =.
В алгебраической форме: =,
в показательной форме: =.
Задача 1.2. Вычислить .
Решение. Представим комплексное число в тригонометрической форме:
=. Воспользуемся следующей формулой извлечения корня из комплексного числа : ), где=0,1,2,…,.
Т.е. корень -ой степени из комплексного числа принимает ровно значений. Точки комплексной плоскости, соответствующие значениям , являются вершинами правильного многоугольника, вписанного в окружность радиуса с центром в начале координат.
В данном примере получаем: =, где . Тогда корни принимают значения:
=(+), =(+),
=(+), =(+).
Для построения этих комплексных
чисел на комплексной плоскости проведем окружность
радиуса . На окружности отметим точку =(+), далее, разбивая окружность на четыре
равные части , изобразим остальные точки ,, . Заметим,
что радиан соответствуют примерно.
Задача 2. Найти произведение матриц , где и .
Решение. ===
Задача 3. Найти матрицу, обратную данной. Сделать проверку. .
Решение. Находим определитель матрицы .
=, т.е. данная матрица является неособенной, обратная матрица существует. Вычислим соответствующие алгебраические дополнения :
==; ==; ;
; ; ;
; ; .
Находим обратную матрицу :
=. Проверка заключается в перемножении матриц
(единичная матрица). Последнюю операцию выполнять аналогично примеру 2.
Задача 4. Решить систему линейных алгебраических уравнений (СЛАУ) с помощью обратной матрицы: .
Решение. Найдем обратную матрицу к матрице системы .
Аналогично примеру 3 получаем =
Вектор – столбец решений находится по формуле:
=, или
Проверка осуществляется непосредственной
подстановкой найденных значений неизвестных в исходную СЛАУ.
Задача 5. Найти ранг матрицы .
Решение. Последовательно осуществляем линейные преобразования строк данной матрицы для приведения ее к ступенчатому виду.
Шаг 1. Переставим в данной матрице первую и вторую строки.
Шаг 2. Умножим первый столбец на 1/2, четвертый столбец умножим на 1/5. В результате получим: .
Шаг 3. Умножим на 2 первую строку и прибавим её к третьей («заработаем» нуль на месте «3-1»,т.е. вместо (–2) получим (0)). В результате получим: .
Шаг 4. Умножим на –3 вторую строку и прибавим её к третьей («заработаем» нуль на месте «3-2»,т.е. вместо (–3) получим (0)). Получим:
Таким образом, получили ступенчатую матрицу эквивалентную данной, в которой две ненулевые строки, значит ее ранг равен 2: .
Задача 6. Решить систему методом Гаусса: .
Решение. Запишем расширенную матрицу СЛАУ:
.
С помощью эквивалентных
преобразований приведём матрицу к верхнетреугольному виду (прямой ход метода
Гаусса).
Шаг1. Вычтем из второй строки первую, результат умножим на (–1/2):
Шаг 2. Вычитаем из третьей строки первую, умноженную на 2:
Шаг 3. Вычитаем из четвёртой строки первую, умноженную на 3:
Шаг 4. Прибавляем к третьей строке вторую, умноженную на 5, результат делим на 2:
Шаг 5. Вычитаем из четвёртой строки вторую:
Шаг 6. Прибавляем к четвёртой строке третью, умноженную на 6, результат делим на 17:
Теперь с помощью эквивалентных преобразований приведём матрицу к диагональному виду (обратный ход метода Гаусса).
Шаги 7, 8, 9. От третьей строки отнимаем четвёртую, умноженную на 2; от второй строки отнимаем четвёртую; от первой строки отнимаем четвёртую.
Шаги 10, 11. От первой строки отнимаем третью; от первой строки отнимаем вторую.
Теперь в последнем столбце
получились искомые значения переменных, т. е.
Задача 7. Найти размерность и базис подпространства решений однородной системы линейных алгебраических уравнений (ОСЛАУ). .
Решение. Применим прямой ход метода Гаусса (приведем систему уравнений к верхнетреугольному виду). Поступая аналогично примеру 6 вычтем из второй строки первую, умноженную на 3; вычтем из третьей строки первую, умноженную на 2; прибавим первую строку к четвёртой; получим:
,
последние три строки оказались одинаковыми, значит, матрицу можно привести к следующему виду:
.
В качестве базисных переменных можно взять , свободные – .
Ранг матрицы равен 2, т.е. , следовательно, размерность подпространства решений тоже равна 2.
Формируем фундаментальную систему решений (ФСР). Пусть переменные принимают значения 1, 0 соответственно. Тогда из второго уравнения находим значение из первого уравнения находим значение Тогда .
Пусть теперь переменные принимают значения 0, 1 соответственно. Тогда, поступая аналогично, получим Откуда
. — образуют
ФСР, общее решение данной однородной СЛАУ имеет вид .
Задача 8. Доказать, что векторы образуют базис и разложить вектор по этому базису.
Решение. Ненулевые векторы образуют базис тогда и только тогда, когда они некомпланарны (т.е. не лежат в одной плоскости), значит их смешанное произведение не должно быть равно 0:
, 121 + 333 + 221 – 223 – 231 – 131 = 12 ≠ 0. Векторы образуют базис.
{2}\end{bmatrix}
.
\]
Прочитать решение
Добавить для решения позже
Линейная алгебра
Задача 720
Найти все собственные значения и соответствующие им собственные векторы матрицы $A$, если
\[
A=
\begin{bmatrix}
2 & -3 & 0 \2 &
-5 & 0 \\
0 & 0 & 3
\end{bmatrix}
.
\]
Прочитать решение
Добавить для решения позже
Линейная алгебра
Задача 719
Пусть $A$ будет матрицей, заданной формулой
\[
A=
\begin{bmatrix}
-2 & 0 & 1 \\
-5 & 3 & a \\
4 & -2 & -1
\end{bmatrix}
\]
для некоторой переменной $a$. Найдите все значения $a$, которые гарантируют, что $A$ имеет собственные значения $0$, $3$ и $-3$.
Прочитать решение
Добавить для решения позже
Линейная алгебра
Задача 718
Пусть
\[
A=
\begin{bmatrix}
8 & 1 & 6 \\
3 & 5 & 7 \\
& 1 2 & 09 9 конец {bmatrix}
.
\]
Обратите внимание, что $A$ содержит все целые числа от $1$ до $9$ и что суммы каждой строки, столбца и диагонали $A$ равны. Такую сетку иногда называют магическим квадратом.
Вычислите определитель $A$.
Чтение решения
\[
T\left(
\begin{bmatrix}
x \\ y
\end{bmatrix}
\right)
=
\begin{bmatrix}
2x+y \\ 0
\end{bmatrix}
,\;
S\left(
\begin{bmatrix}
x \\ y
\end{bmatrix}
\right)
=
\begin{bmatrix}
x+y \\ xy
\end{bmatrix}
.

\] Определить, являются ли $T$, $S$ и композиция $S\circ T$ линейными преобразованиями.
Прочитать решение
Добавить для решения позже
9{3}$ if\[
\mathbf{w}_{1}
=
\begin{bmatrix}
1 \\ 0 \\ 3
\end{bmatrix}
,\quad
\mathbf{w} _{2}
=
\begin{bmatrix}
2 \\ -1 \\ 0
\end{bmatrix}
.
\]
Прочитать решение
Добавить для решения позже
Линейная алгебра
Задача 715
Пусть
\[
\mathbf{v}_{1}
=
\begin{bmatrix}
1 \\ 1
\end{bmatrix}
,\;
\mathbf{v}_{2}
=
\begin{bmatrix}
1 \\ -1
\end{bmatrix}
.
\]
Пусть $V=\Span(\mathbf{v}_{1},\mathbf{v}_{2})$. Образуют ли $\mathbf{v}_{1}$ и $\mathbf{v}_{2}$ ортонормированный базис для $V$?
Если нет, то найти ортонормированный базис для $V$.
Прочитать решение
Добавить для решения позже
Линейная алгебра
Задача 714
Пусть $W$ — множество $3\times 3$ кососимметричных матриц. Покажите, что $W$ является подпространством векторного пространства $V$ всех матриц $3\times 3$. Затем покажите остовное множество для $W$. 95$. Определить $m$ и $n$. Кроме того, найдите ранг и ничтожность $A$.
Прочитать решение
Добавить для решения позже
Линейная алгебра
Задача 711
Используя аксиому векторного пространства, докажите следующие свойства.
Пусть $V$ векторное пространство над $\R$. Пусть $u, v, w\in V$.
(a) Если $u+v=u+w$, то $v=w$.
(б) Если $v+u=w+u$, то $v=w$.
(c) Нулевой вектор $\mathbf{0}$ уникален.
(d) Для каждого $v\in V$ аддитивная обратная $-v$ единственна.
(e) $0v=\mathbf{0}$ для каждого $v\in V$, где $0\in\R$ — нулевой скаляр.
(f) $a\mathbf{0}=\mathbf{0}$ для каждого скаляра $a$.
(g) Если $av=\mathbf{0}$, то $a=0$ или $v=\mathbf{0}$.
(h) $(-1)v=-v$.
Первые два свойства называются закон об отмене .
Прочитать решение
Добавить для решения позже
Линейная алгебра
Задача 710
Найти базис для $\Span(S)$, где $S=
\left\{
\begin{bmatrix}
1 \\ 2 \\ 1
\end {bmatrix}
,
\begin{bmatrix}
-1 \\ -2 \\ -1
\end{bmatrix}
,
\begin{bmatrix}
2 \\ 6 \\ -2
\end{bmatrix }
,
\begin{bmatrix}
1 \\ 1 \\ 3
\end{bmatrix}
\right\}$.
Прочитать решение
Добавить для решения позже
Линейная алгебра
Задача 709
Пусть $S=\{\mathbf{v}_{1},\mathbf{v}_{2},\mathbf{v}_{3},\ mathbf{v}_{4},\mathbf{v}_{5}\}$ где
\[
\mathbf{v}_{1}=
\begin{bmatrix}
1 \\ 2 \\ 2 \\ -1
\end{bmatrix}
,\;\mathbf{v}_{2}=
\begin{bmatrix}
1 \\ 3 \\ 1 \\ 1
\end{bmatrix}
,\ ;\mathbf{v}_{3}=
\begin{bmatrix}
1 \\ 5 \\ -1 \\ 5
\end{bmatrix}
,\;\mathbf{v}_{4}=
\begin{bmatrix}
1 \\ 1 \ \ 4 \\ -1
\end{bmatrix}
,\;\mathbf{v}_{5}=
\begin{bmatrix}
2 \\ 7 \\ 0 \\ 2
\end{bmatrix}
.\]
Найдите базис для пролета $\Span(S)$.
Прочитать решение
Добавить для решения позже
Линейная алгебра
Задача 708
Пусть $A=\begin{bmatrix}
2 & 4 & 6 & 8 \\
1 &3 & 0 & 5 \\
1 & 1 & 6 & 3
\end{bmatrix}$.
(a) Найдите базис для нулевого пространства $A$.
(b) Найдите основу для пространства строк $A$.
(c) Найдите базис для диапазона $A$, состоящего из векторов-столбцов $A$.
(d) Для каждого вектора-столбца, который не является базисным вектором, который вы получили в части (c), выразите его как линейную комбинацию базисных векторов для диапазона $A$. 95$. Если $\mathbf{v}_4$ — другой вектор из $V$, то множество
\[S_2=\{\mathbf{v}_1, \mathbf{v}_2, \mathbf{v}_3, \ mathbf{v}_4\}\]
все еще остовный набор для $V$? Если да, то докажи. В противном случае приведите контрпример.
Прочитать решение
Добавить для решения позже
Линейная алгебра
Задача 705
Для множества $S$ и векторного пространства $V$ над скалярным полем $\K$ определим множество всех функций от $S$ до $V$
\[ \Fun ( S , V ) = \{ f : S \rightarrow V \} . \]
Для $f, g \in \Fun(S, V)$, $z \in \K$ сложение и скалярное умножение могут быть определены как
\[ (f+g)(s) = f(s) + g(s) \, \mbox{ и } (cf)(s) = c (f(s)) \, \mbox{ для всех } s \in S . \]
(a) Докажите, что $\Fun(S, V)$ — векторное пространство над $\K$. Что такое нулевой элемент?
(б) Пусть $S_1 = \{ s \}$ — множество, состоящее из одного элемента. Найдите изоморфизм между $\Fun(S_1 , V)$ и самим $V$. Докажите, что найденная вами карта на самом деле является линейным изоморфизмом. 9m = \{ (v_1 , v_2 , \cdots , v_m ) \mid v_i \in V \mbox{ для всех } 1 \leq i \leq m \} . \]
(e) Используйте базис $B$ в $V$, чтобы построить базис в $\Fun(S, V)$ для произвольного конечного множества $S$. Какова размерность $\Fun(S, V)$?
(f) Пусть $W \subseteq V$ — подпространство. Докажите, что $\Fun(S, W)$ является подпространством в $\Fun(S, V)$.
Прочитать решение
Добавить для решения позже
Линейная алгебра
Задача 704
Пусть $A=\begin{bmatrix}
2 & 4 & 6 & 8 \\
1 & 3 & 0 & 5 \\
1 & 1 & 6 & 3
\end{bmatrix}$ .
(a) Найдите базис для нулевого пространства $A$.
(b) Найдите основу для пространства строк $A$.
(c) Найдите базис для диапазона $A$, состоящего из векторов-столбцов $A$.
(d) Для каждого вектора-столбца, который не является базисным вектором, который вы получили в части (c), выразите его как линейную комбинацию базисных векторов для диапазона $A$.
Прочитать решение
Добавить для решения позже
Линейная алгебра
Задача 703
Используя определение диапазона матрицы, опишите диапазон матрицы
\[A=\begin{bmatrix}
2 & 4 & 1 & -5 \\
1 &2 & 1 & -2 \\
1 & 2 & 0 & -3
\end{bmatrix}.\]
Прочитать решение
Добавить для решения позже
Решение матричных задач по линейной алгебре
Спросил
Изменено 1 год, 6 месяцев назад
Просмотрено 102 раза
$\begingroup$
В классе я узнал, что если вы хотите решать задачи с помощью матрицы
линейной алгебры, задача должна удовлетворять требованиям линейности (однородности, суперпозиции).
Однако уравнения линейных систем типа (x + 2y = 3, 4x + 5y = 6) не удовлетворяют
линейность, потому что они не удовлетворяют требованиям однородности и суперпозиции. Но мы можем
решить эту линейную систему, построив матрицу $$ \begin{pmatrix} 1 и 2 \\ 4 и 5 \\ \end{pmatrix}\begin{pmatrix} Икс\\ у\\ \end{pmatrix} = \begin{pmatrix} 3\\ 6\\ \end{pматрица} $$ и получить обратную матрицу \begin{pmatrix} 1 и 2 \\ 4 и 5 \\ \end{pматрица}. Короче говоря, если я хочу решить проблему с помощью матриц, я узнал, что задача должна удовлетворять линейности, но линейные уравнения не удовлетворяют линейности (суперпозиция, однородность). Так разве не неправильно использовать матрицу для решения задачи линейной системы?
- линейная алгебра
$\endgroup$
2
$\begingroup$
Сначала легко спутать функции и уравнения.
Функция может быть линейной, например $f(x,y) = 2x + 3y$.
Уравнение может быть однородным, например, $f(x,y) = 2x + 3y = 0$, или неоднородным, например, $f(x,y) = 2x + 3y = 7$, но оба они называются линейными. уравнения.
Системы линейных уравнений можно решать с помощью матриц. Если уравнения однородны, то если вы найдете решение, любое постоянное кратное этого решения также является решением. Например, $(-3, 2)$ является решением $2x + 3y = 0$, поэтому $(-30, 20)$ также является решением. Если вы найдете два таких решения, сумма этих решений также является решением.
В более сложной математике некоторые уравнения называются линейными, , если они не содержат постоянных членов, и , аффинными, , если они есть. Они обрабатывают немного более сложные функции .
В некоторых уравнениях, таких как дифференциальные уравнения, в качестве неизвестных переменных используются функции. Эти уравнения называются линейными и однородными, если сумма двух решений является другим решением, а любое постоянное кратное решения является решением. Такая комбинация называется линейной суперпозицией решений. Например (не беспокойтесь обо всех деталях, только о результате): $y» + y = 0$ имеет $y = sin x$ в качестве решения и $y = cos x$ в качестве другого. {-1}
\begin{bmatrix}
3\\
6
\end{bmatrix} =
-\фракция{1}{3}
\begin{bmatrix}
5 и -2\\
-4 и 1
\end{bmatrix}
\begin{bmatrix}
3\\
6
\end{bmatrix} =
\begin{bmatrix}
-1\\
2
\end{bmatrix}
\конец{выравнивание*}
Другими словами, набор решений задается как $S = \{(-1,2)\}$.
В общем случае, учитывая систему линейных уравнений \начать{выравнивать*} \begin{bmatrix} а и б\\ CD \end{bmatrix} \begin{bmatrix} Икс\\ у \end{bmatrix} = \begin{bmatrix} е\\ ф \end{bmatrix} \конец{выравнивание*} где $ad — bc \neq 0$, решение можно записать в виде \начать{выравнивать*} \begin{bmatrix} Икс\\ у \end{bmatrix} = \frac{1}{ad-bc} \begin{bmatrix} д&-б\\ -с и а \end{bmatrix} \begin{bmatrix} е\\ ф \end{bmatrix} \end{align*}
Другой способ интерпретировать такую проблему следующий. 9{2})$.
$\endgroup$
Твой ответ
Зарегистрируйтесь или войдите в систему
Зарегистрируйтесь с помощью Google
Зарегистрироваться через Facebook
Зарегистрируйтесь, используя адрес электронной почты и пароль
Опубликовать как гость
Электронная почта
Требуется, но никогда не отображается
Опубликовать как гость
Электронная почта
Требуется, но не отображается
Нажимая «Опубликовать свой ответ», вы соглашаетесь с нашими условиями обслуживания, политикой конфиденциальности и политикой использования файлов cookie
.
Как машинное обучение использует линейную алгебру для решения проблем с данными
Машины или компьютеры понимают только числа. И эти числа должны быть представлены и обработаны таким образом, чтобы машины могли решать проблемы, обучаясь на данных, а не на заранее определенных инструкциях (как в случае программирования).
Все типы программирования используют математику на том или ином уровне. Машинное обучение включает в себя программирование данных для изучения функции, которая лучше всего описывает данные.
Задача (или процесс) поиска наилучших параметров функции с использованием данных называется модель учебная в ОД.
Таким образом, машинное обучение — это программирование для оптимизации наилучшего возможного решения, и нам нужна математика, чтобы понять, как эта проблема решается.
Первым шагом к изучению математики для машинного обучения является изучение линейной алгебры.
Линейная алгебра — это математическая основа, которая решает проблему представления данных, а также вычислений в моделях машинного обучения.
Это математика массивов — , технически называемых векторами, матрицами и тензорами.
Общие области применения — Линейная алгебра в действии
Источник: https://www.wiplane.com/p/foundations-for-data-science-mlВ контексте МО все основные этапы разработки модели имеют линейный алгебра работает за кулисами.
Важными областями применения, доступными благодаря линейной алгебре, являются:
- представление данных и изученной модели
- встраивание слов
- уменьшение размерности
представление данных
Топливо моделей машинного обучения, то есть данные , необходимо преобразовать в массивы, прежде чем вы сможете передать их в свои модели. Вычисления, выполняемые с этими массивами, включают такие операции, как умножение матриц (скалярное произведение). Это дополнительно возвращает выходные данные, которые также представлены в виде преобразованной матрицы/тензора чисел.
Вложения слов
Не беспокойтесь о терминологии здесь — речь идет только о представлении крупноразмерных данных (подумайте об огромном количестве переменных в ваших данных) с помощью вектора меньшей размерности.
Обработка естественного языка (NLP) работает с текстовыми данными. Работа с текстом означает понимание значения большого корпуса слов. Каждое слово представляет другое значение, которое может быть похоже на другое слово. Векторные вложения в линейной алгебре позволяют нам более эффективно представлять эти слова.
Собственные векторы (SVD)
Наконец, такие понятия, как собственные векторы, позволяют нам уменьшить количество признаков или измерений данных, сохраняя при этом суть всех из них, используя нечто, называемое Анализ главных компонентов.
От данных к векторам
Источник: https://www.
Линейная алгебра в основном имеет дело с векторами и матрицами (различными формами массивов) и операциями над ними массивы. В NumPy векторы в основном представляют собой одномерный массив чисел, но геометрически они имеют как величину, так и направление.
Источник: https://www.wiplane.com/p/foundations-for-data-science-mlНаши данные могут быть представлены с помощью вектора. На рисунке выше одна строка в этих данных представлена вектором признаков, который имеет 3 элемента или компонента, представляющие 3 разных измерения. N-элементы в векторе делают его n-мерным векторным пространством, и в этом случае мы можем видеть 3-мерное пространство.
Глубокое обучение — Тензоры, протекающие через нейронную сеть
Сегодня мы видим линейную алгебру в действии во всех основных приложениях. Примеры включают анализ настроений в сообщениях LinkedIn или Twitter (встраивание), обнаружение типа легочной инфекции по рентгеновским изображениям (компьютерное зрение) или любой бот преобразования речи в текст (NLP).
Все эти типы данных представлены числами в тензорах. Мы запускаем векторизованные операции, чтобы изучать из них закономерности с помощью нейронной сети. Затем он выводит обработанный тензор, который, в свою очередь, декодируется для получения окончательного вывода модели.
Каждая фаза выполняет математические операции над этими массивами данных.
Уменьшение размерности — Преобразование векторного пространства
Источник: https://www.wiplane.com/p/foundations-for-data-science-ml заменяется другим вектором, принадлежащим пространству меньшей размерности. Это более значимо, и это то, что преодолевает вычислительные сложности.Например, вот трехмерный вектор, замененный двумерным пространством. Но вы можете экстраполировать это на сценарий реального мира, где у вас есть очень большие числовые измерения.
Уменьшение измерений не означает удаление функций из данных. Вместо этого речь идет о поиске новых функций, которые являются линейными функциями исходных функций и сохранении дисперсии исходных функций.
Нахождение этих новых переменных (признаков) приводит к нахождению основных компонентов (ПК). Затем это сходится к решению проблем собственных векторов и собственных значений.
Механизмы рекомендаций — Использование вложений
Вы можете думать о встраивании как о 2D-плоскости, встроенной в 3D-пространство, и отсюда появился этот термин. Вы можете думать о земле, на которой вы стоите, как о двухмерной плоскости, встроенной в это пространство, в котором вы живете.
Просто чтобы дать вам реальный пример использования, связанный со всем этим обсуждением встраивания векторов, все приложения, которые дают вам персональные рекомендации, используют встраивание векторов в той или иной форме.
Например, выше приведен график из курса Google по системам рекомендаций, где нам предоставляются данные о разных пользователях и их любимых фильмах. Некоторые пользователи — дети, а другие — взрослые, некоторые фильмы стали классикой на все времена, а другие — более художественными. Некоторые фильмы ориентированы на более молодую аудиторию, в то время как такие фильмы, как сувениры, предпочитают взрослые.
Теперь нам нужно не только представить эту информацию в числах, но и найти новые векторные представления меньшего размера, которые хорошо охватывают все эти функции.
Очень быстрый способ понять, как мы можем справиться с этой задачей, — понять что-то, называемое матричной факторизацией, которая позволяет нам разбивать большую матрицу на более мелкие матрицы.
Пока не обращайте внимания на числа и цвета и просто попытайтесь понять, как мы разбили одну большую матрицу на две меньшие.
Например, здесь эта матрица 4X5, 4 строки и 5 функций была разбита на две матрицы, одна 4X2, а другая 2X5. В основном у нас есть новые векторы меньшего размера для пользователей и фильмов.
И это позволяет нам изобразить это в двумерном векторном пространстве. Здесь вы увидите, что пользователь №1 и фильм «Гарри Поттер» ближе, а пользователь №3 и фильм «Шрек» ближе.
Концепция скалярного произведения (умножения матриц) векторов говорит нам больше о сходстве двух векторов. И у него есть приложения для расчета корреляции / ковариации, линейной регрессии, логистической регрессии, PCA, сверток, PageRank и многих других алгоритмов.
Отрасли, в которых линейная алгебра широко используется
Надеюсь, вы уже убедились, что сегодня линейная алгебра является движущей силой инициатив машинного обучения во многих областях. Если нет, вот список, чтобы назвать несколько:
- Статистика
- Химическая физика
- геномика
- Word Embeddings — Нейронные сети/глубокое обучение
- Physitums
- . Алгебру нужно знать, чтобы начать работу с ML/DL?
Теперь важный вопрос заключается в том, как научиться программировать эти концепции линейной алгебры. Ответ заключается в том, что вам не нужно изобретать велосипед, вам просто нужно понять основы векторной алгебры с помощью вычислений, а затем вы научитесь программировать эти концепции с помощью NumPy.
NumPy — это пакет научных вычислений, который дает нам доступ ко всем основным понятиям линейной алгебры. Он быстрый, так как запускает скомпилированный код C и имеет большое количество математических и научных функций, которые мы можем использовать.
Рекомендуемые ресурсы
- Плейлист по линейной алгебре от 3Blue1Brown — очень увлекательные визуализации, объясняющие суть линейной алгебры и ее приложений. Может быть слишком сложно для начинающих.
- Книга по глубокому обучению Яна Гудфеллоу и Йошуа Бенджио — фантастический ресурс для изучения машинного обучения и прикладной математики. Прочтите его, мало кто может счесть его слишком техническим и сложным для начала.
Основы науки о данных и машинного обучения — Я создал курс, который дает вам достаточное представление о программировании, математике (базовая алгебра, линейная алгебра и исчисление) и статистике.
Полный пакет для первых шагов в изучении DS/ML.
👉 Вы можете использовать код
FREECODECAMP10
, чтобы получить скидку 10%.Ознакомьтесь с планом курса здесь:
Научитесь программировать бесплатно. Учебная программа freeCodeCamp с открытым исходным кодом помогла более чем 40 000 человек получить работу в качестве разработчиков. Начать
Линейная алгебра Приложение C | University of Wisconsin River Falls
Все курсы профессионального образования, ведущие к сертификации, должны включать преподавание и оценку Стандартов содержания штата Висконсин в области содержания.
В этом столбце перечислите стандарты штата Висконсин, которые включены в этот курс.
908:20Стандарты для каждой области контента можно найти в документе Wisconsin Content Standards.
В этой колонке укажите характер оценок успеваемости, используемых в этом курсе для оценки уровня владения учащимися каждым стандартом.
Структуры внутри дисциплины, исторические корни и развивающаяся природа математики, а также взаимодействие между технологией и дисциплиной.
Учащиеся используют аксиоматическую структуру для доказательства результатов о матрицах и векторных пространствах. Учащиеся используют графические калькуляторы или систему компьютерной алгебры для вычисления определителей и решений линейных систем уравнений. *
Содействие формированию у учащихся концептуального и процедурного понимания.
Учащиеся демонстрируют понимание линейной алгебры с алгебраической и геометрической точек зрения.
Учащиеся демонстрируют понимание процедур, выполняя вычисления с линейными системами, и демонстрируют концептуальное понимание, используя свойства этих процедур для решения задач о линейных системах.*
Помощь всем учащимся в формировании понимания дисциплины, включая:
. Уверенность в своих способностях использовать математические знания.
. Осознание полезности математики.
. Экономические последствия тонкой математической подготовки.
Учащиеся объединяют свои знания из предыдущих курсов по алгебре с новыми методами работы с матрицами и векторными пространствами, которые они изучают, и обобщают вычислительные процедуры для доказательства общих результатов, обретая уверенность в своих способностях использовать математические знания. *
Исследование, предположения, изучение и тестирование всех аспектов решения проблем.
Учащиеся делают и проверяют предположения, определяя, являются ли заданные множества векторными пространствами и являются ли заданные функции линейными преобразованиями.
*
Формулирование и постановка математических задач, решение проблем с использованием нескольких стратегий, оценка результатов, обобщение решений, эффективное использование подходов к решению проблем и применение математического моделирования к реальным ситуациям.
Учащиеся обобщают вычисления и вычислительные процедуры, чтобы доказать обобщенные результаты о линейных пространствах и преобразованиях.
Создание убедительных математических аргументов, постановка математических вопросов и предположений, формулирование контрпримеров, построение и оценка аргументов, а также использование интуитивного, неформального исследования и формального доказательства.
Учащиеся пишут математические аргументы и строят контрпримеры по темам линейной алгебры, включая установление тождеств и идентификацию векторных пространств и линейных преобразований.
*
Выражение идей устно, письменно и визуально с использованием математического языка, обозначений и символов; перевод математических идей между контекстами и между ними.
Учащиеся представляют свои математические аргументы визуально и в письменной форме, используя математический язык и символы. Учащиеся представляют векторные пространства малой размерности как геометрически, так и алгебраически, и осуществляют перевод между этими контекстами. *
Соединение понятий и процедур математики, установление связей между математическими направлениями, между математикой и другими дисциплинами, а также с повседневной жизнью.
Учащиеся используют не только процедуру вычислений в линейных системах, но также используют концепции свойств этих вычислений для решения задач (например, свойства редукции строк используются в рассуждениях об определителях и линейной независимости).
Студенты демонстрируют знания о связях между геометрией и алгеброй в своей работе с векторными пространствами малой размерности. *
Выбор подходящих представлений для облегчения решения математических задач и перевод между представлениями для разъяснения ситуаций решения задач.
Учащиеся решают задачи на перевод геометрических и алгебраических форм векторных пространств. Студенты используют алгебраические, геометрические и обобщенные формы представления векторов и линейных преобразований в математических рассуждениях. *
Математические процессы в том числе:
. Решение проблем.
. Коммуникация.
. Рассуждения и формальные и неформальные аргументы.
. Математические связи.
. Представительства.
. Технологии.
Учащиеся решают задачи, требующие соотнесения конкретных примеров с обобщенным контекстом и структурой аксиом.
Учащиеся приводят формальные аргументы по вопросам линейной алгебры. Студенты математически сообщают свои результаты; используя алгебраические, геометрические и обобщенные представления. Учащиеся используют графические калькуляторы или систему компьютерной алгебры для вычисления определителей и решений линейных систем уравнений. *
Числовые операции и отношения как с абстрактной, так и с конкретной точек зрения, идентифицирующие приложения реального мира, а также представляющие и связывающие математические концепции и процедуры, включая:
. Чувство числа.
. Теория множеств.
. Количество и действие.
. Составление и разложение чисел, включая разрядное значение, простые числа, множители, кратные, обратные значения и распространение этих понятий на всю математику.
. Системы счисления через действительные числа, их свойства и отношения.
. Вычислительные процедуры.
. Пропорциональное рассуждение.
. Теория чисел.
Учащиеся используют свои навыки числовых и алгебраических вычислений при решении задач линейной алгебры. Студенты изучают операции над матрицами, в том числе обратные матрицы. *
Математические понятия и процедуры, а также связи между ними для обучения числовым операциям и отношениям верхнего уровня, включая:
. Расширенные процедуры подсчета, включая объединение и пересечение наборов, а также операции со скобками.
. Алгебраические и трансцендентные числа.
. Комплексная система счисления, включая полярные координаты.
. Приближенные методы как основа для численного интегрирования, фракталов и численных доказательств.
. Ситуации, в которых числовые аргументы представлены в различных ситуациях в классе и в реальном мире (например, политических, экономических, научных, социальных), могут быть созданы и подвергнуты критическому анализу.
. Возможности, с помощью которых можно оценить допустимые пределы погрешности (например, оценка стратегий, проверка обоснованности результатов и использование технологий для выполнения вычислений).
Учащиеся выполняют операции с матрицами и векторами и приводят математические аргументы в отношении результатов операций; это способствует более высокому пониманию учащимися концепций чисел и операций посредством исследования этой похожей системы. Этот класс не затрагивает напрямую ни одну из перечисленных подтем. *
Геометрия и измерения как с абстрактной, так и с конкретной точек зрения, а также для определения реальных приложений, а также математических концепций, процедур и связей между ними, включая:
. Формальный и неформальный спор.
. Названия, свойства и взаимосвязи двух- и трехмерных фигур.
. Пространственное чувство.
. Пространственное мышление и использование геометрических моделей для представления, визуализации и решения проблем.
. Преобразования и способы, которыми вращение, отражение и перемещение фигур могут иллюстрировать понятия, свойства и отношения.
. Системы координатной геометрии, включая отношения между координатной и синтетической геометрией, а также обобщение геометрических принципов от двухмерной системы к трехмерной системе.
. Понятия измерения, включая измеримые атрибуты, стандартные и нестандартные единицы измерения, точность и правильность, а также использование соответствующих инструментов.
. Структура систем измерения, включая разработку и использование систем измерения и отношения между различными системами. Измерение, включая длину, площадь, объем, размер углов, вес и массу, время, температуру и деньги.
. Измерение, оценка и использование измерений для описания и сравнения геометрических явлений.
. Косвенное измерение и его использование, включая разработку формул и процедур для определения меры для решения проблем.
Учащиеся используют методы линейной алгебры для анализа линий, плоскостей и пространств с точки зрения вектора, в том числе отношений измерения углов и перпендикулярности.
Учащиеся изучают повороты, отражения и расширения в контексте линейных преобразований и переводят геометрическую и матричную форму этих преобразований. Учащиеся используют векторные системы координат в двух- и трехмерных измерениях и, в более общем случае, в более высоких измерениях. Учащиеся связывают некоторые понятия синтетической геометрии со свойствами векторных координат. Учащиеся используют матричные и векторные методы для измерения длины, площади и объема заданных вектором фигур (в частности, отрезков прямых, параллелограммов и параллелепипедов). *
Математические понятия, процедуры и связи между ними для обучения геометрии и измерениям верхнего уровня, включая:
. Системы геометрии, включая евклидову, неевклидову, координатную, трансформационную и проективную геометрию.
. Преобразования, координаты и векторы и их использование при решении задач. Трехмерная геометрия и ее обобщение на другие измерения. Топология, включая топологические свойства и преобразования.
. Возможность представить убедительные аргументы посредством демонстрации, неформального доказательства, контрпримеров или других логических средств, чтобы показать истинность утверждений и/или обобщений.
Учащиеся используют координатную геометрию в векторной перспективе. Студенты применяют, анализируют и описывают линейные преобразования, векторы, координаты и системы координат. Учащиеся решают задачи и пишут математические рассуждения о преобразованиях, координатах, системах координат и векторах. Учащиеся используют векторы и матрицы для анализа и описания линий, плоскостей и подпространств в двух, трех и более измерениях. Студенты создают и делятся математическими аргументами, доказательствами и контрпримерами в контексте векторных пространств и линейных преобразований. *
Статистика и вероятность как с абстрактной, так и с конкретной точки зрения, а также для определения реальных приложений, а также математических концепций, процедур и связей между ними, включая:
.
Использование данных для изучения реальных проблем.
. Процесс исследования, включающий постановку проблемы, разработку плана сбора данных, а также сбор, запись и организацию данных.
. Представление данных в виде графиков, таблиц и сводной статистики для описания распределения данных, центральной тенденции и дисперсии.
. Анализ и интерпретация данных.
. Случайность, выборка и вывод.
. Вероятность как способ описания шансов или риска в простых и сложных событиях.
. Прогнозирование результатов на основе экспериментов или теоретических вероятностей.
Не оценивается в этом курсе.
Математические понятия, процедуры и связи между ними для обучения статистике и вероятности верхнего уровня в том числе:
. Использование случайной величины при генерации и интерпретации вероятностных распределений.
. Описательная и выводная статистика, показатели выплат, включая достоверность и надежность, и корреляция.
. Теория вероятностей и ее связь со статистикой вывода.
. Дискретные и непрерывные распределения вероятностей как основы для вывода.
. Ситуации, в которых учащиеся могут анализировать, оценивать и критиковать методы и выводы статистических экспериментов, о которых сообщается в журналах, журналах, средствах массовой информации, рекламе и т. д.
Не оценивается в этом курсе.
Функции, алгебра и основные понятия, лежащие в основе исчисления как с абстрактной, так и с конкретной точки зрения, и для определения реальных приложений, а также математические понятия, процедуры и связи между ними, включая:
. Узоры.
. Функции, используемые для описания отношений и моделирования реальных ситуаций.
. Представления ситуаций, которые включают переменные величины с выражениями, уравнениями и неравенствами и которые включают алгебраические и геометрические отношения.
. Множественные представления отношений, сильные стороны и ограничения каждого представления и преобразование из одного представления в другое.
. Атрибуты полиномиальных, рациональных, тригонометрических, алгебраических и экспоненциальных функций.
. Операции над выражениями и решение уравнений, систем уравнений и неравенств конкретными, неформальными и формальными методами.
. Основные концепции исчисления, включая скорость изменения, пределы и приближения для неправильных областей.
Учащиеся используют геометрические свойства (параллельные или перпендикулярные подпространства или проекции на подпространства) и связывают их с соответствующими векторными вычислениями, используя выражения и уравнения. Учащиеся демонстрируют знания о линейных преобразованиях и их атрибутах как функциях в векторных пространствах. Учащиеся переводят стандартные и векторные формы уравнений для линий и плоскостей. Учащиеся решают и используют системы линейных уравнений с использованием матричных методов.
*
Математические понятия, процедуры и связи между ними для обучения функциям верхнего уровня, алгебре и понятиям исчисления, включая:
. Понятия исчисления, включая пределы (эпсилон-дельта) и касательные, производные, интегралы, последовательности и ряды.
. Моделирование для решения проблем.
. Методы исчисления, включая нахождение пределов, производных, интегралов и использование специальных правил.
. Приложения для вычислений, включая моделирование, оптимизацию, скорость и ускорение, площадь, объем и центр масс.
. Численные и приближенные методы, включая правило Симпсона, правило трапеций, приближение Ньютона и линеаризацию.
. Многомерное исчисление.
. Дифференциальные уравнения.
Не оценивается в этом курсе.
Дискретные процессы как с абстрактной, так и с конкретной точек зрения, а также для определения реальных приложений, а также математических концепций, процедур и связей между ними, включая:
.
Техники счета.
. Представление и анализ задач дискретной математики с использованием последовательностей, теории графов, массивов и сетей.
. Итерация и рекурсия.
Не оценивается в этом курсе.
Математические понятия, процедуры и связи между ними для обучения дискретной математике верхнего уровня в том числе:
. Темы, включая символическую логику, индукцию, линейное программирование и конечные графы.
. Матрицы как математическая система, а матрицы и матричные операции как средства записи информации и решения задач.
. Разработка и анализ алгоритмов.
Учащиеся демонстрируют знания о матрицах в вычислительном отношении и в контексте векторного пространства.
*Учащиеся демонстрируют свои знания в этих областях на уроках, домашних заданиях, контрольных или экзаменационных задачах.
Линейная алгебра — Вопросы с решениями
Вопросы линейной алгебры с решениями и подробными объяснениями по матрицам, пространствам, подпространствам и векторам, определителям, системам линейных уравнений и онлайн-калькуляторы линейной алгебры включены.
Поддержите этот веб-сайт, отправив подарок через Paypal и используя мой адрес электронной почты [email protected]
Матрицы
Матрицы с примерами и вопросы с решениями.
Транспонирование матрицы.
Симметричная матрица.
Матрица идентичности.
Диагональные матрицы.
Треугольная матрица.
Диагонализация матриц.
Вопросы по обратной матрице с решениями.
Найти обратную матрицу с помощью операций со строками
Матрицы сложения, вычитания и скалярного умножения.
Умножение и мощность матриц
вопросов о собственных значениях и собственных векторах с решениями, включая два видео на Найдите собственные векторы и собственные значения матрицы 3 на 3 на видео и найдите собственные векторы и собственные значения матрицы 2 на 2 на видео
Операции со строками и элементарные матрицы.
Операции с тремя строками над расширенными матрицами .
Напишите матрицу в сокращенной эшелонированной форме строк.
Повороты матрицы в форме эшелона строк — примеры с решениями.
Пустое пространство и недействительность матрицы .
Пространства столбцов и строк и ранг матрицы .
Свободные и основные переменные матрицы — примеры с решениями.
Ортогональные матрицы — примеры с решениями.
QR-разложение матрицы.
LU Разложение матрицы.
Свойства матричных операций .Задачи наименьших квадратов
Решайте задачи наименьших квадратов с помощью нормальных уравнений.
Решение задач наименьших квадратов с помощью QR-разложения.Пространства, подпространства и векторы
Векторные пространства — примеры с решениями.
Подпространства — Примеры с решениями.
Векторы в ℝ n .
Внутренний продукт, ортогональность и длина векторов .
Ортогональные векторы — примеры с решениями .
Линейные комбинации и размах векторов.
Линейно независимые и зависимые векторы — примеры с решениями .
Проверка линейности векторов в подпространстве — примеры с решениями .
Базис, координаты и размерность векторных пространств .
Изменение основы — примеры с решениями .
Ортонормированный базис — примеры с решениями .
Процесс Грама Шмидта для ортонормированного базиса . Примеры с решениямиопределители
Определитель квадратной матрицы.
Найдите определитель с помощью сокращения строк.Системы линейных уравнений
Три элементарные операции над системами.
Исключение Гаусса для решения систем — вопросы с решениями.
Метод исключения в системах — вопросы с решениями.
Правило Крамера с вопросами и решениями.Видео по линейной алгебре
Найдите собственные векторы и собственные значения матрицы 2 на 2 на видео.
Решите систему уравнений 2 на 2 методом исключения на видео.Калькуляторы линейной алгебры
Найдите обратную матрицу с помощью редукции строк.
Калькулятор формы эшелона строк .
Сокращение строк расширенных матриц — калькулятор .
Онлайн-калькулятор определителя матрицы любого размера .
QR Калькулятор разложения матриц .
Online LU Декомпозиция матричного калькулятора.
Решатель систем уравнений с комплексными коэффициентами.2.5: Применение линейных уравнений
- Последнее обновление
- Сохранить как PDF
- Идентификатор страницы
- 18337
- Аноним
- LibreTexts
- Определять ключевые слова и фразы, переводить предложения в математические уравнения и разрабатывать стратегии решения задач.
- Решите текстовые задачи на отношения между числами.
- Решить геометрические задачи с периметром.
- Решайте процентные и денежные задачи, включая простые проценты.
- Постановка и решение задач равномерного движения.
- Шаг 1 : Прочитайте задачу несколько раз, определите ключевые слова и фразы и систематизируйте предоставленную информацию.
- Шаг 2 : Определите переменные, назначив букву или выражение неизвестным величинам.
- Шаг 3 : Переведите и составьте алгебраическое уравнение, моделирующее проблему.
- Шаг 4 : Решите полученное алгебраическое уравнение.
- Шаг 5 : Наконец, ответьте на вопрос в форме предложения и убедитесь, что он имеет смысл (проверьте).
- Ответить
Целые числа: \(27, 29\) и \(31\).
- Ответить
Ширина: \(8\) футов; длина: \(15\) футов
- Ответить
$\(3600\) инвестируется по \(3,5\)% и $\(2300\) по \(4,5\)%.
- Ответить
Кейт требуется \(15\) минут, чтобы наверстать упущенное.
- Упростите процесс решения реальных задач, создав математические модели, описывающие отношения между неизвестными. Используйте алгебру для решения полученных уравнений.
- Угадывать и проверять решения — плохая практика. Этот метод может иногда давать правильные ответы, но он ненадежен, особенно когда проблемы становятся более сложными.
- Прочитайте задачу несколько раз и найдите ключевые слова и фразы. Определите неизвестные и назначьте переменные или выражения неизвестным величинам. Ищите отношения, которые позволяют вам использовать только одну переменную. Создайте математическую модель ситуации и используйте алгебру для решения уравнения. Проверьте, имеет ли решение смысл, и представьте его в виде предложения.
- Не избегайте текстовых задач: их решение может быть увлекательным и полезным. С большим количеством практики вы обнаружите, что они на самом деле не так уж и плохи. Моделирование и решение приложений — одна из основных причин изучения алгебры.
- Не расстраивайтесь, если первая попытка решения словесной задачи не сработала. Это часть процесса. Попробуйте что-то другое и учитесь на неправильных попытках.
- Сумма числа и \(6\) равна \(37\).
- Если из некоторого числа дважды вычесть \(12\), получится \(6\).
- Число, умноженное на четырнадцать меньше, чем \(5\), равно \(1\).
- Дважды из \(30\) вычитается некоторое число, и в результате получается \(50\).
- Пятикратная сумма \(6\) и некоторого числа равна \(20\).
- Сумма \(5\), умноженная на некоторое число и \(6\), равна \(20\).
- Если из \(10\) вычесть сумму числа и \(3\), получится \(5\).
- Сумма произведений, умноженных на три числа и на то же число в пять раз, равна \(24\).
- Из некоторого числа дважды вычитается десять, и в результате получается сумма числа и \(2\).
- Шесть меньше некоторого числа в десять раз больше суммы этого числа и \(5\).
- Ответить
1. \(х+6=37\)
3. \(5x−14=1\)
5. \(5(х+6)=20\)
7. \(10−(x+3)=5\)
9. \(2x−10=x+2\)
- Большее целое число \(1\) более чем в два раза превышает другое целое число. Если сумма целых чисел равна \(25\), найдите целые числа.
- Если большее целое число в \(2\) больше, чем в \(4\) раз другого целого числа, и их разница составляет \(32\), найдите целые числа.
- Одно целое число на \(30\) больше другого целого числа. Если разница между большим и удвоенным меньшим равна \(8\), найдите целые числа.
- Частное некоторого числа и \(4\) равно \(22\). Найдите число.
- Восемь раз число уменьшается на одно и то же число в три раза, что дает разницу в \(20\).
Какой номер?
- Одно целое число на две единицы меньше другого. Если их сумма равна \(−22\), найдите два целых числа.
- Сумма двух последовательных целых чисел равна \(139\). Найдите целые числа.
- Сумма трех последовательных целых чисел равна \(63\). Найдите целые числа.
- Сумма трех последовательных целых чисел равна \(279\). Найдите целые числа.
- Разница удвоенного меньшего из двух последовательных целых чисел и большего равна \(39\). Найдите целые числа.
- Если меньшее из двух последовательных целых чисел вычесть из удвоенного большего, то получится \(17\). Найдите целые числа.
- Сумма двух последовательных четных целых чисел равна \(46\). Найдите целые числа.
- Сумма двух последовательных четных целых чисел равна \(238\). Найдите целые числа.
- Сумма трех последовательных четных целых чисел равна \(96\). Найдите целые числа.
- Если меньшее из двух последовательных четных целых чисел вычесть из \(3\) раз, тем больше будет результат \(42\).
Найдите целые числа.
- Сумма трех последовательных четных целых чисел равна \(90\). Найдите целые числа.
- Сумма двух последовательных нечетных целых чисел равна \(68\). Найдите целые числа.
- Сумма двух последовательных нечетных целых чисел равна \(180\). Найдите целые числа.
- Сумма трех последовательных нечетных целых чисел равна \(57\). Найдите целые числа.
- Если меньшее из двух последовательных нечетных целых чисел вычесть из удвоенного большего, получится \(23\). Найдите целые числа.
- Удвоенная сумма двух последовательных нечетных целых чисел равна \(32\). Найдите целые числа.
- Разница между удвоенным большим из двух последовательных нечетных целых чисел и меньшим из них составляет \(59\). Найдите целые числа.
- Ответить
1. \(8, 17\)
3. \(22, 52\)
5. \(4\)
7. \(69, 70\)
9.
\(92, 93, 94\)
11. \(15, 16\)
13. \(118, 120\)
15. \(18, 20\)
17. \(33, 35\)
19. \(17, 19, 21\)
21. \(7, 9\)
- Если периметр квадрата равен \(48\) дюймов, найдите длину каждой стороны.
- Длина прямоугольника на \(2\) дюйма больше, чем его ширина. Если периметр равен \(36\) дюймов, найдите длину и ширину.
- Длина прямоугольника на \(2\) фута меньше его ширины более чем в два раза. Если периметр равен \(26\) футов, найдите длину и ширину.
- Ширина прямоугольника на \(2\) сантиметра меньше половины его длины. Если периметр равен \(56\) см, найдите длину и ширину.
- Длина прямоугольника на \(3\) фута меньше его ширины более чем в два раза. Если периметр равен \(54\) футов, найдите размеры прямоугольника.
- Если длина прямоугольника в два раза больше ширины, а его периметр равен \(72\) дюймам, найдите размеры прямоугольника.
- Периметр равностороннего треугольника равен \(63\) сантиметрам. Найдите длину каждой стороны.
- Периметр равнобедренного треугольника, длина основания которого составляет половину длины двух других равных сторон, равен \(25\) сантиметрам. Найдите длину каждой стороны.
- Каждая из двух равных сторон равнобедренного треугольника в два раза длиннее основания. Если периметр равен \(105\) сантиметров, то какой длины каждая сторона?
- В треугольнике есть стороны, меры которых представляют собой последовательные четные целые числа. Если периметр равен \(42\) дюймам, найдите длину каждой стороны.
- В треугольнике есть стороны, меры которых представляют собой последовательные нечетные целые числа. Если периметр равен \(21\) дюймам, найдите размер каждой стороны.
- В треугольнике есть стороны, меры которых являются последовательными целыми числами.
Если периметр равен \(102\) дюймов, то найдите размер каждой стороны.
- Длина окружности равна \(50π\) единиц. Найдите радиус.
- Длина окружности равна \(10π\) единиц. Найдите радиус.
- Длина окружности равна \(100\) сантиметрам. Определить радиус с точностью до десятых.
- Длина окружности равна \(20\) сантиметрам. Найдите диаметр, округлив его до сотых.
- Диаметр круга составляет \(5\) дюймов. Определить длину окружности с точностью до десятых.
- Диаметр круга равен \(13\) футам. Вычислить точное значение окружности.
- Ответить
1. \(12\) дюймов
3. Ширина: \(5\) футов; длина: \(8\) футов
5. Ширина: \(10\) футов; длина: \(17\) футов
7. \(21\) см
9. \(21\) сантиметр, \(42\) сантиметр, \(42\) сантиметр
11. \(5\) дюймов, \(7\) дюймов, \(9\) дюймов
13. \(25\) шт.
15. \(15,9\) см
17. \(15,7\) дюймов
- Рассчитайте простой процент, полученный от \(2\)-летней инвестиции в размере $\(1550\) при годовой процентной ставке \(8\frac{3}{4}\)%.
- Рассчитайте простой процент, полученный от \(1\)-летней инвестиции в размере $\(500\) при годовой процентной ставке \(6\)%.
- На сколько лет нужно инвестировать $\(10 000\) по годовой процентной ставке \(8\frac{1}{2}\)%, чтобы получить $\(4 250\) в виде простых процентов?
- На сколько лет нужно инвестировать $\(1000\) под \(7,75\)% годовых, чтобы получить $\(503,75\) в виде простых процентов?
- Под какую годовую процентную ставку нужно вложить \(2500$) на \(3\) лет, чтобы получить $\(412,50\) в виде простых процентов?
- Под какую годовую процентную ставку нужно вложить \(500$) на \(2\) лет, чтобы получить $\(93,50\) в виде простых процентов?
- Если простые проценты, полученные за \(1\) год, составляли $\(47,25\), а годовая ставка составляла \(6,3\)%, какова была основная сумма долга?
- Если простые проценты, полученные за \(2\) лет, составляли $\(369,60\), а годовая ставка составляла \(5\frac{1}{4}\)%, какова была основная сумма долга?
- Джо вложил налоговую декларацию за прошлый год \(2500\) долларов на два разных счета.
Он положил большую часть денег на счет денежного рынка, зарабатывая \(5\)% простых процентов. Остальное он вложил в компакт-диск, заработав \(8\)% простых процентов. Сколько он положил на каждый счет, если общая сумма процентов за год составила $\(138,50\)?
- Джеймс вложил $\(1600\) на два счета. Один счет зарабатывает \(4,25\)% простых процентов, а другой зарабатывает \(8,5\)%. Если проценты через \(1\) год составили $\(85\), сколько он вложил в каждый счет?
- Джейн вложила свои сбережения в $\(5400\) на два счета. У нее есть часть его на компакт-диске под \(3\)% годовых, а остальная часть на сберегательном счете, который приносит \(2\)% годовых. Если простые проценты, полученные с обоих счетов, составляют $\(140\) за год, то сколько у нее есть на каждом счете?
- Марти разместил прошлогодний бонус в размере $\(2400\) на два счета. Он вложил часть в CD с \(2,5\)% годовых, а остальную часть в фонд денежного рынка с \(1,3\)% годовых. Его общая сумма процентов за год составила $\(42,00\).
Сколько он вложил в каждый счет?
- Алиса кладет деньги на два счета, один с годовой процентной ставкой \(2\)%, а другой с годовой процентной ставкой \(3\). Она инвестирует в \(3) раза больше в счет с более высокой доходностью, чем в счет с более низкой доходностью. Если ее общая сумма процентов за год составляет $\(27,50\), сколько она вложила в каждый счет?
- Джим вложил наследство в два разных банка. Один банк предлагал \(5,5\)% годовых, а другой \(6\frac{1}{4}\)%. Он вложил в более доходный банковский счет вдвое больше, чем в другой. Если его общие простые проценты за \(1\) год составляли $\(4860\), то какова была сумма его наследства?
- Если товар рекламируется по цене $\(29,99\) плюс \(9,25\)% налога, какова общая стоимость?
- Если товар рекламируется как стоимость $\(32,98\) плюс налог \(8\frac{3}{4}\)%, какова общая стоимость?
- Товар, включая налог \(8,75\)%, стоит $\(46,49\). Какова первоначальная стоимость товара до вычета налогов?
- Товар, включая налог \(5,48\)%, стоит $\(17,82\).
Какова первоначальная стоимость товара до вычета налогов?
- Если стоимость обеда составляет $\(32,75\), какова общая сумма после добавления \(15\)% чаевых?
- Сколько чаевых составляет \(15\)% от счета в ресторане, который составляет $\(33,33\)?
- У Рэя есть несколько десятицентовиков и пятицентовиков стоимостью $\(3,05\). У него \(5\) больше десятицентовиков, чем пятаков. Сколько у него каждой монеты?
- У Джилл \(3\) меньше полдолларов, чем четвертаков. Стоимость всех \(27\) ее монет составляет $\(9,75\). Сколько каждой монеты у Джилл?
- Кэти должна внести $\(410\) пяти- и десятидолларовых банкнот. У нее \(1\) меньше чем в три раза больше десятков, чем пятидолларовых банкнот. Сколько штук каждой купюры она должна положить на депозит?
- У Билли есть стопка четвертаков, десятицентовиков и пятаков стоимостью $\(3,75\). У него \(3\) больше десятицентовиков, чем четвертаков, и \(5\) больше пятицентовиков, чем четвертаков.
Сколько каждой монеты у Билли?
- У Мэри есть кувшин с однодолларовыми купюрами, полудолларовыми монетами и четвертаками стоимостью $\(14.00\). У нее в два раза больше четвертаков, чем у полдолларовых монет, и столько же полдолларовых монет, сколько у однодолларовых банкнот. Сколько у нее каждого?
- У Чада есть банкноты в один, пять и десять долларов на общую сумму $\(118\). У него \(2\) более чем в \(3\) раз больше, чем у него пятидолларовых банкнот, и \(1\) меньше десятидолларовых, чем пятидолларовых. Сколько штук каждой купюры у Чеда?
- Ответить
1. $\(271,25\)
3. \(5\) лет
5. \(5,5\)%
7. $\(750.00\)
9. Джо вложил $\(2,050\) на счет денежного рынка и $\(450\) в CD.
11. У Джейн есть $\(3200\) на компакт-диске и $\(2200\) на сбережениях.
13. Алиса вложила $\(250\) под \(2\)% и $\(750\) под \(3\)%.
15.
$\(32,76\)
17. $\(42,75\)
19. $\(37,66\)
21. У него \(17\) пятаков и \(22\) десятицентовиков.
23. У Кэти есть \(12\) пятерок и \(35\) десятидолларовых купюр.
25. У Мэри есть \(7\) однодолларовых банкнот, \(7\) полудолларовых монет и \(14\) четвертаков.
- Две машины выезжают из одного места и едут в противоположных направлениях. Если одна машина в среднем развивает \(55\) миль в час, а другая — в среднем \(65\) миль в час, то сколько времени потребуется им, чтобы преодолеть расстояние в \(300\) миль?
- Два самолета вылетели из аэропорта одновременно в противоположных направлениях. Средние скорости самолетов составляют \(450\) миль в час и \(395\) миль в час. Через какое время самолеты окажутся на расстоянии \(1478,75\) миль друг от друга?
- Билл и Тед мчатся по стране.
Билл уходит на \(1\) час раньше, чем Тед, и едет со средней скоростью \(60\) миль в час. Если Тед намеревается догнать его со скоростью \(70\) миль в час, то сколько времени это займет?
- Два брата уезжают из одного места, один на машине, а другой на велосипеде, чтобы встретиться в доме своей бабушки за ужином. Если один брат в среднем проезжает \(30\) миль в час на машине, а другой в среднем \(12\) миль в час на велосипеде, то брату на велосипеде требуется \(1\) час меньше, чем \(3 \) раз дольше, чем другой в машине. Сколько времени потребуется каждому из них, чтобы совершить путешествие?
- Пилот коммерческой авиакомпании летел со средней скоростью \(350\) миль в час, прежде чем ему сообщили, что аэродром его пункта назначения может быть закрыт из-за плохих погодных условий. Пытаясь успеть до шторма, он увеличил скорость \(400\) миль в час и летел еще \(3\) часа. Если общее пройденное расстояние было \(2,950\) миль, то сколько времени заняла поездка?
- Два брата проехали \(2793\) мили из Лос-Анджелеса в Нью-Йорк.
Один из братьев, ведя машину днем, разгонялся в среднем до \(70) миль в час, а другой, ведя машину ночью, разгонялся в среднем до \(53) миль в час. Если брат, ехавший ночью, проехал на \(3\) часа меньше, чем брат, ехавший днем, то сколько часов проехал каждый из них?
- Джо и Эллен живут \(21\) миль друг от друга. Отправляясь одновременно, они движутся навстречу друг другу. Если у Джо в среднем \(8\) миль в час, а у Эллен в среднем \(6\) миль в час, через сколько времени они встретятся?
- Если дорога до автомастерской занимает \(6\) минут со средней скоростью \(30\) миль в час, то сколько времени потребуется, чтобы идти обратно со средней скоростью \(4\) миль в час?
- Джейме и Алекс покидают одно и то же место и движутся в противоположных направлениях. Условия дорожного движения позволили Алексу в среднем на \(14\) миль в час быстрее, чем Хайме. Через \(1\frac{1}{2}\) часов они находятся на расстоянии \(159\) миль друг от друга. Найдите скорость, с которой мог двигаться каждый из них.
- Джейн и Холли живут \(51\) миль друг от друга и уезжают в одно и то же время, направляясь навстречу друг другу, чтобы встретиться за обедом. Джейн ехала по автостраде со скоростью, в два раза превышающей среднюю скорость Холли. Они смогли встретиться через полчаса. С какой скоростью ехал каждый?
- Ответить
1. \(2,5\) часа
3. \(6\) часов
5. \(8\) часов
7. \(1\frac{1}{2}\) часов
9. Хайме: \(46\) миль в час; Алекс: \(60\) миль в час
- Обсудите в уме идеи расчета налогов и чаевых.
- Исследуйте исторические методы представления неизвестных.
- Найдите и сравните простые и сложные проценты. В чем разница?
- Обсудите, почему алгебра является обязательным предметом.
- Найдите способы показать, что повторяющаяся десятичная дробь является рациональной.
Цели обучения
Ключевые слова, перевод и стратегия
Алгебра упрощает процесс решения реальных задач. Это делается путем использования букв для обозначения неизвестных, переформулирования задач в виде уравнений и предложения систематических методов решения этих уравнений. Чтобы решить задачи с помощью алгебры, сначала переведите формулировку задачи в математические утверждения, описывающие взаимосвязь между данной информацией и неизвестными. Обычно этот перевод в математические утверждения является трудным шагом в этом процессе. Ключом к переводу является внимательное прочтение задачи и выявление определенных ключевых слов и фраз.
Ключевые слова | Перевод |
---|---|
Сумма , умножить на, более чем, плюс, добавить, всего | \(+\) |
Разница , уменьшить на, вычесть из, меньше, минус | \(-\) |
Продукт , умножить на, из, раз, дважды | \(*\) |
Частное , деленное на отношение, на | \(/\) |
Is , итог, результат | \(=\) |
Вот несколько примеров переведенных ключевых фраз.
Ключевые фразы | Перевод |
---|---|
Сумма числа и 7. | \(х+7\) |
Семь больше, чем номер. | |
Разница числа и 7. | \(х-7\) |
Семь меньше числа. | |
Семь вычесть из номер. | |
Продукт из 2 и номер. | \(2x\) |
Дважды номер.![]() | |
Половина от номера . | \(\ гидроразрыва{1}{2}х\) |
Частное числа и 7. | \(\ гидроразрыва {х} {7}\) |
При переводе предложений в математические выражения обязательно прочитайте предложение несколько раз и определите ключевые слова и фразы.
Пример \(\PageIndex{1}\)
Перевести:
Четыре меньше чем удвоенное некоторое число равно \(16\).
Решение :
Сначала выберите переменную для неизвестного числа и определите ключевые слова и фразы. Пусть \(x\) представляет неизвестное, обозначенное «некоторым числом».
Рисунок \(\PageIndex{1}\)
Помните, что вычитание не является коммутативным. По этой причине будьте осторожны при настройке различий. В этом примере \(4−2x=16\) является неправильным переводом.
Ответ :
\(2x−4=16\)
Важно сначала определить переменную — пусть x представляет… — и выразить словами, что такое неизвестная величина. Этот шаг не только сделает вашу работу более читабельной, но и заставит вас задуматься о том, что вы ищете. Обычно, если вы знаете, что вас просят найти, то задача найти это достижима.
Пример \(\PageIndex{2}\)
Перевести:
Когда \(7\) вычитается из \(3\), умноженной на сумму числа и \(12\), получается \( 20\).
Решение :
Пусть \(n\) представляет неизвестное число.
Рисунок \(\PageIndex{2}\)
Ответ :
\(3(n+12)−7=20\)
нужны скобки структуры следующих двух предложений и их переводы:
“ 3 раза сумма числа и 12” | \(3(n+12)\) |
«сумма 3 числа и 12» | \(3n+12\) |
Ключ в том, чтобы сосредоточиться на фразе « 3 раза больше суммы ». Это побуждает нас сгруппировать сумму в круглых скобках, а затем умножить на 3. Как только приложение будет преобразовано в алгебраическое уравнение, решите его, используя методы, которые вы изучили.
Рекомендации по составлению и решению задач Word
А пока настройте все свои уравнения, используя только одну переменную. Избегайте двух переменных, ища взаимосвязь между неизвестными.
Проблемы, связанные с отношениями между действительными числами
Мы классифицируем приложения, связанные с отношениями между действительными числами, в широком смысле, как задачи с числами. Эти проблемы иногда можно решить, используя творческую арифметику, догадки и проверки. Решение таким образом не является хорошей практикой, и его следует избегать. Начните с выполнения основных шагов, описанных в общих рекомендациях по решению текстовых задач.
Пример \(\PageIndex{3}\)
Большее целое число \(2\) меньше, чем меньшее целое число, умноженное на \(3\). Сумма двух целых чисел равна \(18\). Найдите целые числа.
Решение :
Идентификация переменных : Начните с присвоения переменной меньшего целого числа.
Пусть \(x\) представляет меньшее целое число.
Используйте первое предложение, чтобы определить большее целое число в терминах переменной \(x\): «Большее целое число равно 2 меньше чем 3 раза меньше ».
Пусть \(3x-2\) представляет большее целое число.
Составьте уравнение : Сложите выражения, представляющие два целых числа, и установите полученное выражение равным \(18\), как указано во втором предложении: « сумма двух целых чисел равна \(18\)».
\(x+(3x-2)=18\)
Решите : Решите уравнение, чтобы получить меньшее целое число \(x\).
\(\begin{align} x+(3x-2)&=18 \\ x+3x-2&=18 \\ 4x-2&=18 \\ 4x-2\color{Cerulean}{+2}&= 18\color{Cerulean}{+2} \\ 4x&=20 \\ \frac{4x}{\color{Cerulean}{4}}&=\frac{20}{\color{Cerulean}{4}} \ \ x&=5 \end{aligned}\)
Обратная замена : Используйте выражение \(3x−2\), чтобы найти большее целое число — это называется обратной заменой.
\(3x-2=3(\color{OliveGreen}{5}\color{black}{)-2=15-2=13}\)
Ответьте на вопрос : Два целых числа: \(5\) и \(13\).
Чек : \(5 + 13 = 18\). Ответ имеет смысл.
Пример \(\PageIndex{4}\)
Разница между двумя целыми числами равна \(2\). Большее целое \(6\) меньше меньшего более чем в два раза. Найдите целые числа.
Решение :
Используйте соотношение между двумя целыми числами во втором предложении: «Большее целое число 6 меньше , чем вдвое меньше », чтобы идентифицировать неизвестные с точки зрения одной переменной.
Пусть \(x\) представляет меньшее целое число.
Пусть \(2x-6\) представляет большее целое число.
Поскольку разница положительна, вычтите меньшее целое число из большего.
\((2x-6)-x=2\)
Решить.
\(\begin{align} \color{OliveGreen}{2x}\color{black}{-6}\color{OliveGreen}{-x}&=2 \\ x-6&=2 \\ x-6 \color{Cerulean}{+6}&=2\color{Cerulean}{+6} \\ x&=8 \end{aligned}\)
Используйте \(2x − 6\), чтобы найти большее целое число.
\(2x-6=2(\color{Cerulean}{8}\color{black}{)-6=16-6=10}\)
Ответ :
Два целых числа равны \( 8\) и \(10\). Эти целые числа явно решают проблему.
Стоит еще раз напомнить, что часто можно найти решение простых задач, угадывая и проверяя. Это так, потому что числа выбраны для упрощения процесса решения, чтобы алгебраические шаги не были слишком утомительными. Вы научитесь составлять алгебраические уравнения с более простыми задачами, чтобы потом использовать эти идеи для решения более сложных задач.
Пример \(\PageIndex{5}\)
Сумма двух последовательных четных целых чисел равна \(46\). Найдите целые числа.
Решение :
Ключевая фраза, на которой следует сосредоточиться, — «последовательные четные целые числа».
Пусть \(x\) представляет первое четное целое число.
Пусть \(x+2\) представляет следующее четное целое число.
Сложите четные целые числа и установите их равными \(46\).
\(x+(x+2)=46\)
Решить.
\(\begin{aligned}\color{OliveGreen}{x+x}\color{black}{+2}&=46\\2x+2&=46\\2x+2\color{Cerulean}{- 2}&=46\color{Cerulean}{-2}\\2x&=44\\x&=22 \end{выровнено}\)
Используйте \(x + 2\), чтобы найти следующее четное целое число.
\(x+2=\color{Cerulean}{22}\color{black}{+2=24}\)
Ответ :
Последовательные четные целые числа равны \(22\) и \( 24\).
Должно быть ясно, что последовательные четные целые числа разделены двумя единицами. Однако может быть не так ясно, что и нечетные целые числа тоже.
Рисунок \(\PageIndex{3}\)
Пример \(\PageIndex{6}\)
Сумма двух последовательных нечетных целых чисел равна \(36\). Найдите целые числа.
Решение :
Ключевая фраза, на которой следует сосредоточиться, — «последовательные нечетные целые числа».
Пусть \(x\) представляет первое нечетное целое число.
Пусть \(x+2\) представляет следующее нечетное целое число.
Сложите два нечетных целых числа и установите выражение равным \(36\).
\(x+(x+2)=36\)
Решить.
\(\begin{aligned} \color{OliveGreen}{x+x}\color{black}{+2}&=36 \\ 2x+2&=36 \\ 2x+2\color{Cerulean}{- 2}&=36\color{Cerulean}{-2} \\ 2x&=34 \\ \frac{2x}{\color{Cerulean}{2}}&=\frac{34}{\color{Cerulean}{ 2}} \\ x&=17 \end{выровнено}\)
Используйте \(x + 2\), чтобы найти следующее нечетное целое число.
\(x+2=\color{OliveGreen}{17}\color{black}{+2=19}\)
Ответ :
Последовательные нечетные целые числа равны \(17\) и \( 19\).
Алгебраическая установка для задач с четными и нечетными целыми числами одинакова. Распространенной ошибкой является использование \(x\) и \(x + 3\) при идентификации переменных для последовательных нечетных целых чисел. Это неверно, потому что прибавление 3 к нечетному числу дает четное число: например, \(5 + 3 = 8\). Неправильная настройка, скорее всего, приведет к десятичному ответу, что может указывать на то, что проблема была настроена неправильно.
Пример \(\PageIndex{7}\)
Сумма трех последовательных целых чисел равна \(24\). Найдите целые числа.
Решение :
Последовательные целые числа разделены одной единицей.
Пусть \(x\) представляет первое целое число.
Пусть \(x+1\) представляет следующее целое число.
Пусть \(x+2\) представляет собой третье целое число.
Сложите целые числа и установите сумму, равную \(24\).
\(x+(x+1)+(x+2)=24\)
Решить.
\(\begin{aligned} \color{OliveGreen}{x+x}\color{black}{+1}\color{OliveGreen}{+x}\color{black}{+2}&=24\\ 3x+3&=24 \\ 3x+3\color{Cerulean}{-3}&=24\color{Cerulean}{-3} \\ 3x&=21 \\ x&=7 \end{aligned}\)
Обратно подставьте, чтобы найти два других целых числа.
\(x+1=\color{OliveGreen}{7}\color{black}{+1=8}\)
\(x+2=\color{OliveGreen}{7}\color{black} {+2=9}\)
Ответ :
Три последовательных целых числа равны \(7, 8\) и \(9\), где \(7 + 8 + 9 = 24\).
Упражнение \(\PageIndex{1}\)
Сумма трех последовательных нечетных целых чисел равна \(87\). Найдите целые числа.
Задачи по геометрии (периметр)
Напомним, что периметр многоугольника представляет собой сумму длин всех внешних ребер. Кроме того, полезно просмотреть следующие формулы периметра \((π≈3,14159)\).
Периметр прямоугольника: | \(P=2l+2w\) |
Периметр квадрата: | \(P=4s\) |
Периметр треугольника: | \(Р=а+b+с\) |
Периметр круга (длина окружности): | \(С=2\пи г\) |
Имейте в виду, что вы ищете взаимосвязь между неизвестными, чтобы можно было составлять алгебраические уравнения, используя только одну переменную. При работе с задачами по геометрии часто бывает полезно нарисовать рисунок.
Пример \(\PageIndex{8}\)
Периметр прямоугольника равен \(64\) футам. Длина в \(4\) фута больше, чем в \(3\) раза больше ширины. Найдите размеры прямоугольника.
Решение :
Предложение «Длина на 4 фута больше , чем в 3 раза больше ширины » дает отношение между двумя переменными.
Рисунок \(\PageIndex{4}\)
Пусть \(w\) обозначает ширину прямоугольника.
Пусть \(3w+4\) представляет длину.
Предложение «Прямоугольник имеет периметр измерений \(64\) футов» предполагает алгебраическую установку. Подставьте \(64\) вместо периметра и выражения для длины в соответствующую формулу следующим образом:
\(\begin{aligned} P&=\:\:\:\:\quad 2l + 2w \\ \color {Cerulean}{\downarrow}&\:\:\:\qquad\quad\color{Cerulean}{\downarrow} \\ \color{OliveGreen}{64}&=2(\color{OliveGreen}{3w+4 }\color{черный}{)+2w} \end{выровнено}\)
После того, как вы составили алгебраическое уравнение с одной переменной, найдите ширину \(w\).
\(\begin{align} 64&=\color{OliveGreen}{6w}\color{black}{+8+}\color{OliveGreen}{2w} \\ 64&=8w+8 \\ 64\color{ Cerulean}{-8}&=8w+8\color{Cerulean}{-8} \\ 56&=8w \\ \frac{56}{\color{Cerulean}{8}}&=\frac{8w}{ \color{Cerulean}{8}} \\ 7&=w \end{aligned}\)
Используйте \(3w + 4\), чтобы найти длину.
\(l=3w+4=3(\color{OliveGreen}{7}\color{black}{)+4=21+4=25}\)
Ответ :
Размеры прямоугольника \(7\) футов на \(25\) футов. Для проверки сложите все стороны:
\(P=7\text{ft+}7\text{ft+}25\text{ft+}25\text{ft}=64\text{ft}\)
Пример \(\PageIndex{9}\)
Две стороны треугольника на \(5\) и \(7\) дюймов длиннее третьей стороны. Если периметр равен \(21\) дюймам, найдите длину каждой стороны.
Рисунок \(\PageIndex{5}\)
Решение :
Первое предложение описывает отношения между неизвестными.
Пусть \(x\) обозначает длину третьей стороны.
Пусть \(x+5\) и \(x+7\) представляют длины двух других сторон.
Подставьте эти выражения в соответствующую формулу и используйте \(21\) для периметра \(P\).
\(\begin{align} P&=a+b+c \\ \color{OliveGreen}{21}&=\color{OliveGreen}{x}\color{black}{+}\color{OliveGreen}{ (x+5)}\color{black}{+}\color{OliveGreen}{(x+7)} \end{выровнено}\)
Теперь вам нужно решить уравнение с одной переменной.
\(\begin{align} 21&=x+x+5+x+7 \\ 21&=3x+12 \\ 21\color{Cerulean}{-12}&=3x+12\color{Cerulean}{ -12} \\ 9&=3x\\ \frac{9}{\color{Cerulean}{3}}&=\frac{3x}{\color{Cerulean}{3}} \\ 3&=x \end{ выровнено}\)
Обратный заменитель.
\(x+5=\color{OliveGreen}{3}\color{black}{+5=8}\)
\(x+5=\color{OliveGreen}{3}\color{black} {+7=10}\)
Ответ :
Три стороны треугольника имеют размеры \(3\) дюймов, \(8\) дюймов и \(10\) дюймов. Чек остается читателю.
Упражнение \(\PageIndex{2}\)
Длина прямоугольника на \(1\) фут меньше, чем его ширина в два раза. Если периметр равен \(46\) футов, найдите размеры.
Задачи, связанные с деньгами и процентами
Всякий раз, когда составляется уравнение с процентами, нам обычно нужно преобразовать проценты в десятичные дроби. Если вопрос требует процента, то не забудьте перевести свой ответ в процент в конце. Кроме того, когда речь идет о деньгах, не забудьте округлить до двух знаков после запятой.
Пример \(\PageIndex{10}\)
Если пара обуви стоит $\(52,50\), включая налог \(7\frac{1}{4}\)%, какова первоначальная стоимость товар до начисления налогов?
Решение :
Начните с преобразования \(7\frac{1}{4}\)% в десятичное число.
\(7\frac{1}{4}%=7,25%=0,0725\)
Сумма налога равна этой ставке, умноженной на первоначальную стоимость товара. Первоначальная стоимость товара – это то, что вас просят найти.
Пусть \(c\) представляет стоимость товара, \(\underline{\text{до налогов}}\) добавляется.
\(\color{Cerulean}{сумма\:налог\:=\:налог\:ставка\:\cdot\:стоимость\:из\:предмет}\)
\(=0,0725\cdot c\)
\(\color{Cerulean}{всего\:стоимость\:=\:стоимость\:из\:предмета\:+\:сумма\:из\:налог}\)
\(52,50= c+0,0725c\)
Используйте это уравнение, чтобы найти \(c\), первоначальную стоимость товара.
\(\begin{align} 52. 50&=\color{OliveGreen}{1c+0.0725c} \\ 52.50&=1.0725c \\ \frac{52.50}{\color{Cerulean}{1.0725}}&=\ frac{1.0725c}{\color{Cerulean}{1.0725}} \\ 48,95& \приблизительно c \end{выравнивание}\)
Ответ :
Стоимость товара до вычета налогов составляет $\(48,95\). Проверьте это, умножив $\(48,95\) на \(0,0725\), чтобы получить налог и добавить его к этой стоимости.
Пример \(\PageIndex{11}\)
Учитывая годовую процентную ставку \(5\frac{1}{8}\)%, сколько времени понадобится $\(1200\), чтобы получить доход $\( 307,50\) в виде простых процентов?
Решение :
Пусть \(t\) представляет собой время, необходимое для того, чтобы заработать $\(307,50\) при \(5,125\)%.
Организуйте данные, необходимые для использования формулы простых процентов \(I=prt\).
Проценты за период: | \(I=\)$\(307,50\) |
Данный основной: | \(р=\)$\(1200\) |
Данная ставка: | \(r=5\frac{1}{8}\)%\(=5,125\)%\(=0,05125\) |
Затем подставьте все известные величины в формулу и найдите единственную неизвестную \(t\).
\(\begin{aligned} I&=prt \\ \color{OliveGreen}{307.50}&=\color{OliveGreen}{1200}\color{black}{(}\color{OliveGreen}{0.05125}\color {черный}{)t} \\ 307.50&=61.5t \\ \frac{307.50}{\color{Cerulean}{61.5}}&=\frac{61.5t}{\color{Cerulean}{61.5}} \ \ 5&=t \end{aligned}\)
Ответ :
Требуется \(5\) лет на $\(1200\), вложенных в \(5\frac{1}{8}\)% чтобы заработать $\(307,50\) в виде простых процентов.
Пример \(\PageIndex{12}\)
Мэри вложила все свои сбережения в размере $\(3400\) на два счета. Ее счет взаимных фондов заработал \(8\)% в прошлом году, а ее компакт-диск заработал \(5\)%. Если ее общая сумма процентов за год составила $\(245\), сколько было на каждом счете?
Решение :
Связь между двумя неизвестными такова, что они составляют 3400 долларов. Когда речь идет об общей сумме, распространенный метод, используемый для того, чтобы избежать двух переменных, состоит в том, чтобы представить вторую неизвестную как разность общей суммы и первой неизвестной.
Пусть \(x\) представляет собой сумму, вложенную во взаимный фонд в размере \(8\)%\(=0,08\).
Пусть \(3400-x\) представляет собой оставшуюся сумму, вложенную в компакт-диск в размере \(5\)%\(=0,05\).
Общий процент представляет собой сумму процентов, полученных с каждого счета.
Проценты, полученные в паевом фонде: | \(I=Prt=x⋅0,08⋅1=0,08x\) |
Проценты, полученные в CD: | \(I=Prt=(3400−x)⋅0,05⋅1=0,05(3400−x)\) |
\(\color{Cerulean}{mutual\:fund\:interest\:+\:CD\:interest\:=\:total\:interest}\)
\(0,08x+0,05(3400-x )=245\)
Это уравнение моделирует задачу с одной переменной. Найдите \(х\).
\(\begin{align} 0,08x+0,05(3400-x)&=245 \\ \color{OliveGreen}{0,08x}\color{black}{+170}\color{OliveGreen}{-0,05x }&=245 \\ 0.03x+170\color{Cerulean}{-170} &=245\color{Cerulean}{-170} \\ 0. 03x&=75 \\ \frac{0.03x}{\color{Cerulean }{0.03}}&=\frac{75}{\color{Cerulean}{0.03}} \\ x&=2,500 \end{aligned}\)
Обратная замена.
\(3,400-x=3,400-\color{OliveGreen}{2,500}\color{black}{=900}\)
Ответ :
Мария вложила $\(2,500\) в \(8\ )% во взаимном фонде и $\(900\) на \(5\)% на компакт-диске.
Пример \(\PageIndex{13}\)
У Джо есть несколько десятицентовиков и четвертаков стоимостью $\(5,30\). У него на одну монету меньше, чем в два раза больше, чем четвертаков. Сколько у него каждой монеты?
Решение :
Начните с определения переменных.
Пусть \(q\) представляет количество четвертаков, которые держит Джо.
Пусть \(2q-1\) обозначает количество десятицентовиков.
Чтобы определить общую стоимость нескольких монет, умножьте количество монет на стоимость каждой монеты. Например, \(5\) четвертаков имеют стоимость $\(0,25 ⋅ 5 =\) $\(1,25\).
\(\color{Cerulean}{value\:in\:четверти\:+\:value\:in\:dimes\:=\:total\:value\:of\:coins}\)
\ (0,25q+0,10(2q-1)=5,30\)
Найдите количество кварталов, \(q\).
\(\begin{align} 0,25q+0,10(2q-1)&=5,30 \\ \color{OliveGreen}{0,25q+0,20q}\color{black}{-0,10}&=5,30 \\ 0,45 q-0.10&=5.30 \\ 0.45q-0.10\color{Cerulean}{+0.10}&=5.30\color{Cerulean}{+0.10} \\ 0.45q&=5.40 \\ \frac{0.45q}{\color {Cerulean}{0.45}}&=\frac{5.40}{\color{Cerulean}{0.45}} \\ q&=12 \end{aligned}\)
Обратно подставьте в \(2q − 1\), чтобы найти количество десятицентовиков.
\(2q-1=2(\color{OliveGreen}{12}\color{black}{)-1=24-1=23}\)
Ответ :
Джо имеет \(12\ ) четверти и \(23\) десятицентовика. Проверьте, умножив $\(0,25 ⋅ 12 = \)$\(3,00\) и $\(0,10 ⋅ 23 = \)$\(2,30\). Затем добавьте, чтобы получить правильную сумму: $\(3,00 + \)$\(2,30 = \)$\(5,30\).
Упражнение \(\PageIndex{3}\)
Общая сумма $\(5900\) вложена в два счета. Один счет приносит \(3,5\)% годовых, а другой — \(4,5\)%. Если проценты за \(1\) год составляют $\(229.50\), то сколько вложено в каждый счет?
Задачи равномерного движения (задачи расстояния)
Равномерное движение относится к движению со скоростью или скоростью, которая не изменяется. Мы можем определить пройденное расстояние, умножив среднюю скорость на время, пройденное с этой скоростью, по формуле \(D=r⋅t\). Приложения, связанные с равномерным движением, обычно имеют много данных, поэтому полезно сначала организовать данные в виде диаграммы, а затем составить алгебраическое уравнение, моделирующее проблему.
Пример \(\PageIndex{14}\)
Два поезда отправляются со станции одновременно в противоположных направлениях. Один движется со скоростью \(70\) миль в час, а другой — со скоростью \(60\) миль в час. За какое время расстояние между ними достигнет \(390\) миль?
Решение :
Сначала определите неизвестное количество и систематизируйте данные.
Пусть \(t\) обозначает время, необходимое для того, чтобы разделить \(390\) миль.
Рисунок \(\PageIndex{6}\)
Данная информация заносится в следующую таблицу. Время для каждого поезда равно.
Рисунок \(\PageIndex{7}\)
Чтобы не вводить еще две переменные, используйте формулу \(D=r⋅t\) для заполнения неизвестных расстояний, пройденных каждым поездом.
Расстояние, пройденное поездом 1: \(D=r\cdot t=70\cdot t\)
Расстояние, пройденное поездом 2: \(D=r\cdot t=60\cdot t\)
Теперь мы можем полностью заполнить таблицу.
Рисунок \(\PageIndex{8}\)
Алгебраическая установка определяется столбцом расстояний. Задача заключается в том, чтобы узнать, сколько времени потребуется, чтобы общее расстояние преодолело \(390\) миль.
Рисунок \(\PageIndex{9}\)
Найдите \(t\).
\(\begin{align} 70t+60t&=390 \\ 130t&=390 \\ \frac{130t}{\color{Cerulean}{130}}&=\frac{390}{\color{Cerulean}{130}} \\ t&=3 \end{aligned}\)
Ответ :
Расстояние между поездами достигает \(3\) часов \( 390\) миль.
Пример \(\PageIndex{15}\)
Поезд, идущий без остановок к месту назначения, может двигаться со средней скоростью \(72\) миль в час. На обратном пути поезд делает несколько остановок и в среднем может двигаться только \(48\) миль в час. Если обратный путь занимает на \(2\) часа больше, чем первоначальный путь до пункта назначения, то каково время в пути в каждую сторону?
Решение :
Сначала определите неизвестную величину и систематизируйте данные.
Пусть \(t\) представляет собой время, необходимое для прибытия в пункт назначения.
Пусть \(t+2\) представляет собой время, необходимое для обратного пути.
Рис.0002 Используйте формулу \(D=r⋅t\), чтобы заполнить неизвестные расстояния.
Пройденное расстояние в пункте назначения: \(D=r\cdot t=72\cdot t\)
Пройденное расстояние в обратном направлении: \(D=r\cdot t=48\cdot (t+2)\ )
Используйте эти выражения, чтобы заполнить таблицу.
Рисунок \(\PageIndex{12}\)
Алгебраическая установка снова определяется столбцом расстояний. В этом случае расстояние до пункта назначения и обратно одинаково, а уравнение равно
\(72t=48(t+2)\)
Найдите \(t\).
\(\begin{align} 72t&=48(t+2) \\ 722&=48t+96 \\ 72t-48t&=48t+96-48t \\ 24t&=96 \\ \frac{24t}{24} &=\frac{96}{24} \\ t&=4 \end{aligned}\)
Обратный путь занимает \(t+2=4+2=6\) часов.
Ответ :
Требуется \(4\) часов, чтобы добраться до пункта назначения и \(6\) часов, чтобы вернуться.
Упражнение \(\PageIndex{4}\)
Мэри едет в школу на велосипеде со средней скоростью \(6\) миль в час. Ее сестра Кейт, опаздывая, уезжает на \(15\) минут позже и едет на велосипеде с вдвое большей скоростью. Сколько времени потребуется Кейт, чтобы догнать Мэри? Будь осторожен! Обратите внимание на единицы, указанные в задаче.
Ключевые выводы
Упражнение \(\PageIndex{5}\) Переведите
Переведите следующее в алгебраические уравнения.
Упражнение \(\PageIndex{6}\) Задачи с числами
Составьте алгебраическое уравнение и затем решите его.
Упражнение \(\PageIndex{7}\) Задачи по геометрии
Составьте алгебраическое уравнение и затем решите его.
Упражнение \(\PageIndex{8}\) Задачи на проценты и деньги
Составьте алгебраическое уравнение и затем решите его.
Упражнение \(\PageIndex{9}\) Равномерное движение (задачи на расстояние)
Составьте алгебраическое уравнение и решите его.
Упражнение \(\PageIndex{10}\) Темы на доске обсуждений